SlideShare a Scribd company logo
1 of 141
ONEXAMINATION.COM
What is the diagnosis?
1) Branch retinal vein occlusion
2) Papilloedema
3) Cytomegalovirus retinitis
4) Central retinal artery occlusion
5) Toxoplasma retinitis
The slide shows the typical
'cottage cheese and tomato ketchup' or 'pizza' appearance of
CMV retinitis in a patient with HIV disease
This 60 year-old man presented to his
General Practitioner with a three-
month history of proximal muscle
weakness. He was a smoker of 20
cigarettes a day for 40 years.
From the following list, which would be
the TWO most appropriate
investigations to perform in order to
establish the diagnosis?
1) Serum anti-rnp antibody
2) Serum creatine kinase
3) Skin biopsy
4) 24 hour urinary cortisol excretion
5) Muscle biopsy
6) ACTH stimulation (synachthen) test
7) Uncuffed serum calcium
8) EMG
9) Serum anti-Scl-70 antibody
10) CT scan thorax
The slide shows dermatomyositis. In older patients
presenting for the first time with dermatomyositis an
underlying malignancy should be considered.
Malignancies most commonly associated with
dermatomyositis include lung, ovary, breast and G-I.
The diagnosis is usually made by demonstrating
elevated CK with an abnormal EMG (showing
spontaneous fibrillation). Treatment is with
glucocorticoids, which is often associated with remission
of symptoms, even with an underlying malignancy.
This 42-year-old lady has a history of
repeated bone fractures.
What is the diagnosis?
1) Familial hypercholesterolaemia
2) Gaucher disease
3) Sarcoidosis
4) Phenylketonuria
5) Wilson’s disease
The slide shows yellow papules (pingeculae) in the cornea; these are characteristic of Gaucher disease. Gaucher
disease is inherited as an autosomal recessive disease. Disease is caused by a deficiency of the enzyme
glucocerbrosidase, essentail for the metabolism of glycolipids. There are three types of Gaucher disease:
(1) Type 1 (chronic non-neuropathic; adult Gaucher disease)
(2) Type 2 (acute neuropathic; infentile Gaucher disease)
(3) Type 3 (subacute neuropathic; juvenile Gaucher disease)
Patients with all types of disease have hepatosplenomegaly and large glucocerebroside-rich cells (Gaucher cells)
infiltrating the bone marrow. Type 2 (infentile Gaucher disease) carries the worst prognosis, with children seldom
surviving beyond 2 years. Type 1 disease is the commonest, usually presenting in childhood with
hepatosplenomegaly, but not uncommonly in middle- or old-age. Bone marrow replacement and hypersplenism
result in anaemia and thrombocytopoenia. Pathological bone fractures and avascular necrosis of the femoral heads
are not uncommon. Bony disease may be confined to the distal ends of the femurs, with formation of characteristic
‘Erlenmeyer flask’ shaped cysts. The skin may show a grey-brown discolouration, especially around the forehead,
hands and pre-tibial regions. Characteristic yellow or yellow-brown papules (pingeculae) develop at the
sclerocorneal junctions.
What is the diagnosis?
1)Cutaneous T-cell lymphoma
2)Metastatic malignant melanoma
3)Kaposi's sarcoma
4)Leser-Trelat sign
5)meningiococcal septicaemia
The lesions shown are characteristic of Kaposi's sarcoma (KS).
Lesions can vary in colour: ranging from pink-red-purple-brown;
they can be macular or papular. Large lesions are often surrounded
by an area of ecchymosis; they can be painful of painless.
This 55 year old female is referred with hypertension. She has been hypertensive for over 4 years and control
has been difficult despite a combination of atenolol 50 mg daily, lisinopil 20mg daily and bendrofluazide 2.5 mg
daily. Examination reveals the appearances as shown with a blood pressure of 180/98 mmHg and a pulse
of 66 beats per minute
Which of the following
investigations would be most appropriate?
1) Insulin like growth factor-1
2) Oral glucose tolerance test
3) Random Growth Hormone concentration
4) Renal ultrasound examinationrine
5) Urine catecholamine concentrations
This patient appears acromegalic and
hypertension may be a presenting feature. The
most appropriate investigation would be an oral
glucose tolerance test with GH measurements.
Suppression of growth hormone concentrations
below 2 mU/l would be expected in norml patients
with non-suppression or paradoxical elevation
seen in acromegaly. Elevated IGF-1concentrations
would be expected but this is not diagnostic.
This patient presented following a fall at
home. His ECG on admission showed
first degree heart block.
Which of the following would be most
useful in confirming the diagnosis?
1) EMG
2) Transoesophageal echocardiogram
3) Serum creatine kinase
4) Tensilon test
5) Lyme serology
The slide shows typical facies of myotonic dystrophy, the commonest adult muscular
dysrophy. Features include frontal baldness in men, atrophy of temporalis,
masseters, facial muscle and bilateral ptosis. Neck muscles (including
sternocleidomastoid) are involved early in the course of disease. Cardiac
abnormalities are common, and include first degree heart block and complete heart
block. Myotonic dystrophy is also associated with intellectual impairment, gonadal
atrophy and insulin resistance. Diagnosis can be made on EMG and muscle biopsy.
Which of the following agents is
the most appropriate to treat this
lesion?
1-Gentamicin eye drops
2-Steroid eye drops
3-Ganciclovir ocular implant
4-Aciclovir eye drops
5-Systemic glucocorticoids
Dendritic ulcers are caused by herpes simplex virus. Presentation is usually with
pain, photophobia, blurred vision , conjunctivitis and chemosis. Steroid eye drops
are contraindicated as they may induce massive amoeiboid ulceration and
blindness. Treated with aciclovir eye drops, which should be continued for three
days after the ulcer has healed.
This gentleman is being treated for
ischaemic heart disease and a dilated
cardiomyopathy.
Which of the following agents is most
likely the cause of this appearance?
1-Metoprolol
2-Frusemide
3-Isosorbide mononitrate
4-Lisinopril
5-Amiodarone
The slide shows gynaecomastia. Drugs that induce gynaecomastia include:
inhibitors of testosterone synthesis
(ketoconazole, metronidazole, cimetidine,
etomidate, cisplatin), oestrogens (digitalis, oral contraceptive pill); other drugs in
which the mechanism is not known isoniazid, diazepam, omeprazole, calcium-channel
blockers, ACE inhibitors, tricyclic antidepressants, busulphan, marijuanha, heroin).
The pedigree shown relates to a
disease inherited in an autosomal
recessive manner. What is the chance
of individual 'A' being a carrier of the
abnormal gene?
1) 0%
2) 25%
3) 50%
4) 75%
5) 100%
50%The disease shows an autosomal recessive pattern of
inheritance. For an affected individual to have a child with the
disease, their partner must be a carrier.
This 28 year old female presents
with this appearance of her legs.
What is the diagnosis?
1)Erythema ab igne
2)Erythema nodosum
3)Necrobiosis Lipoidica
Diabeticorum
4)Post-phlebitic leg
5)Pre-tibial myxoedema
This is pretibial myxoedema with the raised, indurated pinkish patches. Can
occur anywhere but typically on the shins and dorsum of feet. Often found with
acropachy and ophthalmopathy and a high titre of TSH recpetor antibodies
would be expected.
A 27 year-old man was referred to
hospital with fevers and
haemoptysis. Two weeks earlier he
had presented to casualty following
a grand mal seizure. Pending the
results of sputum cultures, what is
the most appropriate combination
of antibiotics that should be used to
treat this patient initially?
1)Amoxycillin/clavulanate +
clarithromycin
2)Flucloxacillin
3)Cefuroxime + metronidazole
4)Azithromycin
5)Vancomycin + ceftazidime
The slide shows an abscess in the right mid-zone. The lung abscess is likely the result of
aspiration during the grand mal seizure that occurred several weeks earlier. Due to the
angle of the bronchi, the right lung is more commonly affected by aspiration than the left
lung. Classically, when the subject is lying down, aspirated oral or gastric contents enter
the apical segment of the right lower lobe. Anaerobes and gram negative organisms are
the usual organisms in abscesses following aspiration. Cefuroxime + metronidazole
This patient presented to Casualty
with acute abdominal pain.
What is the diagnosis?
The slide shows pigmentation of the buccal mucosa in
Addison's disease. Addison's disease (primary
hypoadrenalism) commonly presents with non-specific
systemic symptoms, including: abdominal pain;
vomiting; weight loss; diarrhoea; malaise; fever and
muscle cramps.
What is the diagnosis?
1)Pseudoxanthoma elasticum
2)Acanthosis nigricans
3)Intertrigo
4)Neurofibromatosis
5)Addison's disease
Acanthosis nigricans (pictured) has a characteristic
hyperpigmented, velvety surface. It frequently occurs in the
axillae, groins and in the skin fold of the neck and occasionally
on the dorsum of the hand. Acanthosis nigricans is associated
with: endocrine disease (acromegaly, Cushing's syndrome,
insulin resistant diabetes mellitus), in polycystic ovary syndrome
and as a paraneoplastic phenomenon (usually tumours of the GI
tract, especially adenocarcinoma of the stomach).
This 53-year-old woman presented
to hospital with haematemesis. She
had been hospitalised six months
previously following a similar
episode.
What is the diagnosis?
1) Chronic hepatitis B
2) Ehlers-Danlos syndrome
3) Haemachromatosis
4) Pseudoxanthoma elasticum
5) Zollinger-Ellison syndrome
The slide shows 'plucked chicken skin' appearance to skin folds in the neck. Elastic fibres in
the skin degenerate and calcify, leading to this characteristic appearance. Patients are
predisposed to peripheral and coronary arterial disease and to upper and lower
gastrointestinal haemorrhage. Due to loss of elastic tissue, patients with pseudoxanthoma
elasticum have an increased incidence of mitral regurgitation, aortic regurgitation and aortic
dissection.
A 46-year-old man presents with
an extensive pruritic rash shown
in picture A. 2 weeks previously
he had a sore throat with the
appearence shown in picture B.
What is the cause of the rash?
1)Cutaneous T-cell lymphoma
2)Guttate psoriasis
3)Pituriasis Rosea
4)Pityriasis Rubra Pilaris
5)Scarlet fever
Guttate psoriasis takes its name from the appearence of the lesions which are 'drop
like'. It is a common presentation of psoriasis in children and young adults and may
often follow and upper respiratory tract infection especially Streptococcal.Pityriasis
Rosea is of unknown aetiology and is a widespread pink patchy rash that appears
after a 'herald patch'.Pityriasis Rubra Pilaris is a papulosquamous disorder of
unknown aetiology which presents as red-orange plaques. It is a cause of
erythroderma.
A 43 year-old HIV-positive man
presented with this rash. He had been
treated for Pneumocystis carinii
pneumonia (PCP) six months previously
when he presented for the first time. His
current medication included highly active
antiretrovial therapy (zidovudine (AZT),
lamivudine (3TC) and nevirapine) and
co-trimoxazole as PCP prophylaxis. His
CD4 count was 210 cells/mm3 with an
undetectable HIV viral load.
What is the cause of his rash?
1)Co-trimoxazole allergy
2)Disseminated herpes simplex virus
infection
3)Nevirapine hypersensitivity
4)Scabies
5)Varicella zoster virus
The slide shows typical VZV (shingles). HIV-positive patients are
more prone to VZV regardless of their CD4 count. In addition to
the typical dermatomal distribution of the vesicular rash, HIV
patients occasionally have vesicles scattered in adjecent
dermatomes. In advanced HIV disease VZV can manifest as
severe disseminated disease.
This 29 year-old man presented to hospital
with a four week history of progressively
worsening dyspnoea on exertion. He also
complained of a non-productive cough. Over
the two days preceeding admission the
patient had become breathless at rest and
was started on oral co-amoxiclav by his
general practitioner. On examination he was
febrile 38°C and looked unwell. Candida
was noted on the tonsilar pillars. Oxygen
saturation was 95% on room air, but fell to
85% following a bout of coughing. No
wheeze or crackles were heard in his chest.
His chest radiograph is shown. What is the
most likely diagnosis?
1)Pneumococcal pneumonia
2)Chlamydia pneumoniae pneumonia
3)Pneumocystis carinii pneumonia
4)Legionella pneumophilia pneumonia
5)Influenza A pneumonia
The history is typical of PCP. Salient features are: several days/weeks of increasing dyspnoea; dry cough;
marked oxygen desaturation with exercise. The presence of oropharyngeal candidiasis in a patient without a
known immunosuppressive illness is highly suggestive of HIV/AIDS. The very brief course of antibiotics is not
sufficient to explain the presence of Candida.
What is the most likely
visual field defect?
1)Arcuate
2)Central
3)Concentric
4)Lower quadrantic
5)Temporal
The slide shows the typical appearance of toxocara retinitis
with a lesion at the macula. In retinitis due to Toxocara canis,
there is usually only a single, well demarcated lesion.
This 58 year-old gentleman is 6'
6" tall and has small testicles.
What is the likely karyotype?
1) 45 X
2) 46 XY
3) 47 XXX
4) 47 XXY
5) 47 XYY
Picture shows gynaecomastia in a patient with a history suggesting
Kilinefelter's syndrome. Klinefelter's is characterised by tall stature, small
testes, azoospermia and gynaecomastia in a male. Plasma gonadotrophins are
raised. Typical karyotype is 47XXY, though mosaics occur with 46XY/47XXY
karyotype. There is an increased risk of breast cancer (20x higher than normal
male).
This 58 year-old gentleman is 6' 6"
Which of the following
investigation is most likely to
reveal the diagnosis?
1)Testicular biopsy
2)Serum prolactin
3)Buccal smear
4)MRI scan of pituitary and
hypothalamus
5)Plasma testosterone
The slide shows gynaecomastia;
the history (tall stature and small
testes) suggests Klinefelter's
syndrome. Buccal smear will be
chromatin positive, indicating the
presence of an extra X
chromosome.
This 52 year-old Greek woman
presented with a right popliteal
deep vein thrombosis. What is
the most likely diagnosis?
1)Reitter's syndrome
2)Crohn's disease
3)Secondary syphilis
4)Pemphigus vulgaris
5)Behcet's disease
Behcet's disease is a rare multisystem vasculitis commoner in males than females.
Clinical features include the classical triad of (1) recurrent oral ulceration (2)
recurrent genital ulceration (3) iritis. Other features include fever, abdominal pain,
diarrhoea, episcleritis, polyarteritis, arterial and venous thromboses, and pericarditis.
Patients may exhibit pathergy (developnment of pustules at venepuncture sites). HLA
B5 is associated with ocular disease; HLA B12 is associated with recurrent oral ulcers.
Treatment is with steroids or colchicine.
A 48 year-old diabetic man presents with a 24
hour history of an increasingly painful left
leg.
What is the most likely causative organism?
1) Group A Streptococcus
2) Group B Streptococcus
3) Group G Streptococcus
4) Staphylococcus aureus
5) Staphylococcus epidermidis
Ascending cellulitis is shown. Staphylococcus aureus &
Streptococci are the commonest causative organisms.
Group B Streptoccus has a prediliction for diabetic
patients and is the likliest causative organism in this
scenario.
This patient with advanced HIV
disease noted the appearance of
these lesions on her face over
several weeks
What is the causative agent
1) Human papilloma virus
2)Human herpesvirus 8
3) Pox virus
4) Bartonella hensalae
5) Cryptococcus neoformans
The slide shows the typical papules (with central umbilcation) associated
with molluscum contagiosum. The condition is caused by infection with a pox
virus and is seen in advanced HIV disease and AIDS. Bartonella hensalae is
the bacterium that causes cat-scratch disease. Cryptococcus neoformans can
cause cutaneous lesions in advanced HIV disease, but the appearance here is
not typical of cryptococcosis.
A 47 year-old lady presented two
weeks after returning from a
holiday in the Caribbean
complaining of an intensely
pruritic rash on her abdomen.
What is the causative organsim?
1)Ancylostoma braziliense
2)Chironex fleckeri Southcott (Box
Jelly Fish)
3)Borrelia burgdorferi
4)Onchocerca volvulus
5)Treponema pallidum
The figure shows cutaneous larva migrans (creeping eruption). Usually caused by an animal
hookworm, most commonly Ancylostoma braziliense. The infection is acquired by direct contact with
dog or cat faeces – often acquired when sunbathing on contaminated sand, etc. The larvae burrow in
the dermo-epidermal junction. Symptoms include pruritis and a raised, serpiginous erythematous rash
that migrates at a rate of up to 1 cm/day. Treatment is with topical thiobendazole, or oral albendazole.
Acute infection with the human nematodes Strongyloides stercoralis, Necator americanus and
Ancylostoma duodenale, may produce a similar appearance.
A 47 year-old man with advanced HIV
disease presented with fever and
obtundation. He has not attended his
clinic appointments for several years
and is not taking antiretroviral therapy.
A CT scan of his brain was normal. A
lumbar puncture was preformed;
microscopy of his cerbrospinal fluid is
shown below.
What treatment should this patient
receive?
1)Amphotericin B + flucytosine
2)High-dose intravenous cefotaxime
3)Sulfadiazine + pyrimethamine
4)High-dose intravenous aciclovir
5)Highly active antiretroviral therapy.
The slide is an India ink stain of CSF that
shows typical yeast-like forms of
Cryptococcus sp.; the thick
polysaccharide capsule is highlighted
against the darker backgound of black
ink. The history is characteristic of
cryptococcal disease in AIDS. Treatment
is with amphotericin B and flucytosine
(5FC); patients then require lifetime
suppression with fluconazole.
The following 46-year-old man presents with
exertional chest pain after walking about 100
metres on the flat. He is a non-smoker. An
exercise tolerance test confirms the diagnosis
of IHD.
What is the most likely classification of his
hyperlipidaemia?
1) Abetalipoproteinaemia
2) Chylomicronaemia syndrome
3) Familial hypercholesterolaemia
4) Familial hypertriglyceridaemia
5) Remnant hypercholesterolaemia
The picture shows xanthelasma and corneal arcus. These are seen in common 'polygenic'
hypercholesterolaemia, familial combined hyperlipidaemia and familial
hypercholesterolaemia. Tendon xanthomata are seen in familial hypercholesterolaemia and
remnant hypercholesterolaemia. Eruptive xanthomata are seen in chylomicronaemia
syndrome and familial hypertriglyceridaemia. Striate xanthomata (plam creases) are seen in
remnant hypercholesterolaemia.
What is the diagnosis?
1) Erythema multiforme
2) Erythema marginatum
3) Erythema nodosum
4) Erythema gyratum repens
5) Erythema ab igne
The slide shows typical target lesions of erythema multiforme.
These are the shins of a 73 year old female
who is admitted as a consequence of
increasing confusion and inability to look
after herself. Which one of the following
investigations would be most likely to
confirm the underlying diagnosis?
1) CT head scan
2) Plasma Glucose concentration
3) Thyroid function tests
4) Urea and electrolytes
5) Vitamin B12
The diagnosis is erythema ab igne
and this is due to sitting too close
to a fire. It frequently occurs on
the front of the shins or lower
back, the latter especially
associated with the use of a hot
water bottle. In this patient?s case
the confusion and coldness with
erythema ab igne suggest a
diagnosis of hypothyroidism.
This 52 year-old man was found
collapsed on a golf course.
Cardiopulmonary resuscitation
was initiated. On arrival of the
Paramedics, he was noted to be
in ventricular fibrillation.
What was the cause of his
collapse?
1) Acute myocardial infarction
2) Acute rheumatic fever
3) Bacterial endocarditis
4) Cardiac Lyme disease
5) Lightening strike
The figure shows a 'ferning' or
'arborescent' rash pathognomonic of a
lightning strike. Also known as
Lichtenberg figures. "The pathology of
lightning, or keraunopathy, is known only
to a few specialists."
A 47-year-old Turkish man
presented to casualty with this
lesion on his arm. He also
complained of discomfort in the
left axilla. The lesion had
developed over a period of two
weeks and was painless. There
was no past history of note. He
had recently returned from a
vacation to his parents’ goat farm
in Turkey.
What is the diagnosis?
1) Orf
2) Cutaneous anthrax
3) Pyoderma gangrenosum
4) Brucellosis
5) Plague
The slide shows the typical appearance of cutaneous
anthrax, caused by Bacillus anthracis. Cutaneous disease
is the commonest form of the infection in humans and is
usually due to contact with infected animals or animal
products. Anthrax is endemic to herd animals in some
parts of the world. Following exposure, the skin lesion
evolves over a period of ~2 weeks into a papule, pustule,
vesicle and eventually forms an ulcer with a central black
eschar. The surrounding skin is usually boggy and
oedematous. Lesions are usually painless with tender
regional lymph nodes. Lesions heal spontaneously in 80-
90% of cases; 10-20% of patients progress and become
bacteraemic – associated with a high mortality. Penicillin
is effective in treating the infection.
A 45 year-old dockyard worker presented
with symptoms of fatigue and general
malaise. He also reported developing ulcers
on his hands after minor trauma.
What is the most likely diagnosis?
1) Acute intermittent porphyria
2) Alcoholic haemosiderosis
3) Contact dermatitis
4) Haemachromatosis
5) Porphyria cutanea tarda
Porphyria cutanea tarda (PCT) is associated with deficiency of hepatic
uroporphyrinogen (URO) decarboxylase. The major clinical feature is
cutaneous photosensitivity. Bullae develop on sun-exposed areas and lesions
heal slowly, leaving scars. Porphyrins are increased in liver, plasma, urine
and stool. Porphobilinigen (PBG) is normal. Factors contributing to PCT are
alcohol (the commonest cause), excess iron and excess oestrogens.
This 56 year old female presents
with a 6 month history of weight
loss and sweas. Examination
reveals the appearances as shown,
a pulse of 98 beats per minute
and blood pressure is 120/70
mmHg. Investigations show a T4
of 25.3 nmol/l (NR 9.8 - 23), a
TSH of 4.5 mU/l (NR 0.5 - 4.2
mU/l) and a T3 of 7.8 nmol/l (NR
3 - 6).
Which of the following
investigations would be most
appropriate?
1) 123I Thyroid uptake scan
2) Beta-HCG concentration
3) CT ovaries
4) MRI headscan
5) Short synacthen test
This patient has secondary hyperthyroidism with
elevated TSH, T3 and T4. MRI headscan may be
expected to demonstrate a pituitary
macroadenoma. Alpha subunit is also secreted in
large amounts and measurement of this should
yield an elevated alphaSU: TSH ratio (usually
1:1). The diagnosis should be suspected when TSH
concentrations are not suppressed in the presence
of hyperthyroidism.
This patient presented with a three month history of a gradually enlarging rash on her
foot. She had a past history of rheumatoid arthritis. Urine dipstick was negative and a
fasting blood glucose was normal.
What is the most likely diagnosis?
1) Squamous cell carcinoma
2) Myxoedema
3) Neuropathic ulcer
4) Morphoea
5) Necrobiosis lipoidica diabeticorum
Necrobiosis may
prescede symptoms and
signs of diabetes by
several months.
Which of the following statements is correct
regarding this disease?
1) Immunofluorescent staining of a skin biopsy will show
intercellular deposition of IgG at the dermal-epidermal
junction
2) This disease is associated with internal malignancy
3) Faecal porphyrins will be elevated
4) Mucosal involvement is rare
5) This appearance is due to a cutaneous T-cell lymphoma
The slide shows pemphigoid, a skin disease of the elderly (>60 years)
characterised by the development of large blisters that heal without scarring.
The condition is caused by IgG autoantibodies against components of the
basement membrane. Histology shows IgG and C3 deposition along the
basement membrane. Mucosal involvement is rare; if present it is generally
associated with late disease and is seldom present at presentation.
This 52 year old male is referred with
reduced shaving frequency and libido of at
least five years duration. Examination
reveals the appearances as shown and he
is noted to have a bitemporal hemianopia
and slight galactorrhoea to expression.
From the list select the most likely
diagnosis.
1) Acromegaly
2) Klinefelter's syndrome
3) Macroprolactinoma
4) Microprolactinoma
5) Non-functional pituitary tumour
This patient has typical features of hypopituitarism and the galactorrhoea suggests
hyperprolactinaemia. In the presence of optic chiasmal compression the diagnosis is
likely to be a macroprolactinoma. Although stalk compression with a non
functioning tumour may cause hyperprolactinaemia the concentrations of prolactin
are usually below 2000 mU/l and galactorrhoea would be rare. The patient does not
appear acromegalic.
This 82 year-old Jewish lady
presented with this rash.
What is the diagnosis?
1) Toxic epidermal necrolysis
2) Mycosis fungoides
3) Pemphigus vulgaris
4) Porphyria cutanea tarda
5) Pemphigoid
Pemphigus vulgaris is a blistering disease seen in the elderly
and is characterised by the formation of thin-walled blisters.
Manual pressure applied to the skin may induce sloughing
and separation of the epidermis (Nikolsky's sign).
What does the slide show?
1) Intracapillary drusen
2) Grade 1 hypertensive retinopathy
3) Optic atrophy
4) Lipaemia retinalis
5) Papilloedema
Papilloedema
A 15-year-old boy was brought to casualty
by his parents. He had been febrile and
generally unwell for 2 days, but on the day
of presentation had spiked a high fever,
become slightly confused and started
vomiting. A CT head scan was
unremarkable and a lumbar puncture was
performed. A Gram stain of his
cerebrospinal fluid is shown.
What is the diagnosis?
1) Haemophilus influenzae meningitis
2) Listeria meningitis
3) Meningiococcal meningitis
4) Pneumococcal meningitis
5) Viral meningitis
The Gram stains shows Gram-positive
diplococci, characteristic of
Streptococcus pneumoniae.
An 82-year-old man presents with weight loss (5kg) and a hoarse voice of
two months’ duration. His chest radiograph is shown below.
What clinical signs are likely to be found?
1) Dilated left pupil
2) Left ptosis
3) Inability to blink left eyelid
4) Inability to sweat on the right upper body
5) Right-sided exophthalmos
The chest x-ray shows left upper lobar consolidation; the history of hoarseness implies involvement of the
recurrent laryngeal nerve – most likely due to invasion by tumour. Upper lobar malignancies involving the
superior pulmonary sulcus can destroy surrounding structures leading to a characteristic clinical pattern –
Pancoast’s syndrome. The syndrome consists of pain in a C8-T2 distribution (caused by infiltration of these
nerves) often accompanies by radiological evidence of destruction of the first and second ribs. Horner’s
syndrome frequently co-exists due to infiltration of the sympathetic trunk. Horner’s syndrome consists of
enophthalmos, ptosis, miosis and ipsilateral loss of the ability to sweat).
A 27-year-old man with Down syndrome complains to his
carers of intense perianal discomfort and pruritis. He lives
in a long-term care facility and recently completed a
seven-day course of antibiotics for a chest infection.
Examination reveals perianal excoriation. A perianal swab
is sent for microscopy (shown).
What is the cause of his symptoms?
1) Clostridium difficile associated diarrhoea
2) Echinococcus granulosus
3) Enterobius vermicularis
4) Hymenolepis nana
5) Trichuriasis
Enterobius vermicularis (threadworm) infection is not uncommon in children and in institution. Transmission
is by the faeco-oral route and intense anal pruritis is the predominant symptom. Treatment is with
mebendazole or piperazine. Hymenolepsis nana is a rodent cestode parasite that can be transmitted to
humans and usually affects children. Abdominal pain, anorexia, diarrhoea, pruritis ani and urticaria are the
most frequent symptoms. Eosinophilia may be present in heavy infestations. Treatment is with praziquantel.
Echinococcus granulosus is the organism responsible for hydatid disease; it is a canine tapeworm. Trichuriasis
is commoner in malnourished populations; symptoms are minimal but may result in growth retardation in
children. Heavy burdens of infection may be associated with bloody diarrhoea; it is associated with rectal
prolapse. Treatment is with mebendazole.
A 9 year-old boy presents to casualty
with visual disturbance. A visual field
defect is detected on confrontational
testing. Fundoscopy reveals the
appearance shown below.
What is the most likely cause of this
appearance?
1) Cytomegalovirus
2) Medullated nerve fibres
3) Retinoblastoma
4) Toxocara canis
5) Toxoplasma gondii
The slide shows the typical appearance of
toxocara retinitis with a lesion at the macula. In
retinitis due to Toxocara canis, there is usually
only a single, well-demarcated lesion.
What is the diagnosis in this 78 year-old man?
1) Pemphigus vulgaris
2) Dermatitis herpetiformis
3) Toxic epidermal necrolysis
4) Necrolytic migratory erythema
5) Pemphigoid
Pemphigoid is a disease of the elderly (>60 years) characterised by the development
of large blisters that heal without scarring. The condition is caused by IgG
autoantibodies against components of the basement membrane. Blistering in
pemphigoid occurs at the subepidermal level – deeper than the blisters of pemphigus
vulgaris (which occur at the dermal-epidermal junction). Hence the tense blisters
seen in pemphigoid; blisters are thin-walled and fragile in pemphigus – few intact
blisters are ever seen.
A 72 year old woman presents to A&E
with fevers and pain in her right knee.
On examination she is febrile 40°C; her
right knee is swollen, hot and tender. She
is unable to tolerate any passive
movement of the joint. A knee aspirate is
sent for microscopy – the Gram stain is
shown below.
What is the most appropriate
management for this patient?
1) Treat for four weeks with intravenous
benzyl penicillin + flucloxacillin
2) Treat for two weeks with intravenous
vancomycin + rifampicin
3) Start antibiotics once sensitivities are
known
4) Start prednisolone 40mg od
5) Surgical referral
The slide shows Gram positive cocci
growing in pairs and short chains – the
diagnosis is Streptococcal septic
arthritis. Septic arthritis has a high
morbidity and mortality. Although
antibiotics are clearly required, and
should be given without delay, the
patient needs an immediate
orthopaedic surgical referral and
should be taken for urgent joint
washout. The duration of antibiotic
therapy is judged by clinical response,
but will usually be in the order of 4-6
weeks.
This 19-year-old woman presented to the
Accident & Emergency department
complaining of painful legs.
What is the most likely cause for these
lesions?
1) Secondary syphilis
2) Cat scratch fever
3) Oral contraceptive pill
4) Sarcoidosis
5) Streptococcal infection
Erythema nodosum is characterised by the presence of tender subcutaneous nodules,
usually on the shins. Histology of these lesions shows a vasculitis of small venules and
panniculitis. The commonest cause is Streptococcal infection. Other common common
causes include: other upper respiratory tract infections; sarcoidosis, inflammatory bowel
disease. Less common causes: TB, histoplasmosis, coccidioidomycosis, psittacosis, cat
scratch fever, Yersinia infection, Salmonellosis, Chlamydial infection, drugs (oral
contraceptive, sulfonamides, aspartame, bromides, iodides).
This 58 year old lady presented with hypertension. What
clinical sign is being demontrated in picture B?
1) Dehydration
2) Loss of subcutaneous fat
3) Skin fragility
4) Sensory peripheral neuropathy
5) Peripheral oedema
Thin skin and loss of subcutaneous fat is a sign of
Cushing's disease. It should raise the possibility of this
as a diagnosis in individuals presenting with diabetes,
hypertension, hypokalaemia, obesity or purpura.
Cushing disease, the excess secretion of corticotropin
from a pituitary adenoma that stimulates excess
corticosteroid secretion from the adrenal glands, occurs
primarily in women, aged 25 to 45 years. Easy
bruisability is a commonly reported symptom. Orth
and colleagues further describe the indications: "The
skin is atrophic, the stratum corneum is thinned, and
there is loss of subcutaneous fat. The skin becomes
fragile and, in extreme cases, peels off with adhesive
tape like damp tissue paper (Liddle's sign)."
This 33-year-old man presented
with a painful left eye. He was
known to be HIV antibody
positive.
What is the causative agent?
1) Cytomegalovirus
2) Adenovirus
3) Human herpesvirus-8
4) Epstein-Barr virus
5) Pox virus
The slide shows Kaposi's sarcoma (KS) of the eye. A typical KS
lesion is visible on the left side of his forehead. KS can present
with cutaneous or disseminated disease. Lesions on the head
and neck, especially around the eyes, nose and mouth are
common; they may be painful or painless.
This 21 year old female presents with weight loss,
weakness and dizziness.
Which of the following antibodies would provide
diagnostic information?
1) Anti- 21 Hydroxylase antibodies
2) Anti- Glutamic acid decarboxylase antibodies
3) Anti - Intrinsic factor antibodies
4) Anti- Thyroid peroxidase antibodies
5) Anti- Endomysial antibodies
This patient has addison's disease as reflected by the pigmented
appearance and the symptoms. Anti-21 hydroxylase antibodies are
typically seen in a high percentage (80-90%) of such cases. Although other
antibodies may be present, they are not as frequent.
This is the chest X-ray of a 53 year old
engineer presenting with increasing
shortness of breath on exertion.
What is the diagnosis?
1) asbestosis
2) byssinosis
3) siderosis
4) silicosis
5) talcosis
The X-ray shows extensive pleural plaques from previous exposure to
asbestos.
EFFECT
AGENT
DISEASE
Fibrosis, bullae,
pneumothorax
Alum.& al.oxide
Aluminosis
Pleural plaques, lung
cancer,mesothelioma
Asbestos
Asbestosis
Airway obstruction,
loss of elasticity
Cotton, flax, hemp
Byssinosis
Chemical
pneumonitis
Cadmium, Cobalt,
Nickel, Zinc &
others
Metal fume fever
Reversible airway
obstruction
Western Red Cedar
and others
Occupational
Asthma
Dust deposits
Iron oxide
Siderosis
Dust deposits and
fibrosis
Silica
Silicosis
Perivascular fibrosis
Talc, hydrated Mg.
silicates
Talcosis
A 47 year-old man with advanced
HIV disease presented with fever and
obtundation. A CT scan of his brain
was normal. A lumbar puncture was
preformed; microscopy of his
cerbrospinal fluid is shown .
What is the diagnosis?
1) Primary central nervous system
lymphoma
2) Pneumococcal meningitis
3) Toxoplasma encephalitis
4) Syphilitic meningitis
5) Cryptococcal meningitis
Cryptococcus neoformans is a yeast-like fungus that
can cause meningitis in patients with AIDS (CD4 < 50
cells/mm3) and other profoundly immunosuppressed
patients. Symptoms are often mild initially with
headache, slight neck stiffness and fever. However,
patients can become obtunded rapidly. Diagnosis is by
demonstration of C. neoformans in the CSF – shown on
an India ink stain: the thick polysaccharide capsule is
highlighted around the cell. CSF pressures can be very
high – sometimes requiring daily therapeutic CSF
tapping. Treatment is with amphotericin B and
flucytosine (5FC); patients then require lifetime
suppression with fluconazole.
This 50 year old male presents with a three
month history of tiredness, lethargy and
weight gain. Six years previously he
underwent trans-sphenoidal resection of a
non-functional pituitary tumour. He
currently takes hydrocortisone, thyroxine
and testosterone therapy. Examination
reveals the appearances as shown and a BMI
of 30.
Which of the following treatments would you
consider for this patient?
1) Desmopressin
2) DHEAS
3) Fludrocortisone
4) Growth Hormone
5) Sibutramine
This hypopituitary patient complains of tiredeness
and weight gain despite adequate standard
hormone replacement therapy. His current
symptoms appear to be related to a GH deficiency
and would benefit from GH therapy. He does not
require DDAVP as there is no evidence of Diabetes
Insipidus. Fludrocortisone is not required as
adrenal function is preserved and
mineralocorticoid activity which depends on the
renin-angiotensin system and independent of
pituitary function, is normal. Sibutramine which is
licensed for the treatment of obesity would not be
required.
This pattern of inheritance is most
consistent with ...
1) Haemophilia B
2) Hereditary spherocytosis
3) Sickle cell disease
4) Thalassaemia major
5) von Willebrand’s disease
The pedigree chart shows a pattern of X-linked recessive inheritance. Sickle cell
disease and thalassaemias are inherited in an autosomal recessive manner; hereditary
spherocytosis and von Willebrand’s disease are inherited as autosomal dominant
diseases.
Examples of X-linked recessive diseases include:
Haemophilia A & B, Duchenne muscular dystrophy, Becker muscular dystrophy,
glucose-6-phosphate dehydrogenase deficiency, Lesch-Nyhan syndrome, nephrogenic
diabetes insipidus, Hunter’s syndrome, Anderson-Fabry disease, ocular albinism.
What is the diagnosis?
1) Anterior uveitis
2) Dislocation of the lens
3) Hyphaema
4) Hypopyon
5) Malignant melanoma of the iris
The slide shows hyphaema, blood in the anterior chamber. Usually caused by
trauma – often small objects (champagne corks, squash balls) hitting the eye.
Aspiration may be required to prevent loss of vision.
This patient was admitted to
hospital with severe central
abdominal pain.
What name is given to this
appearance?
1) Tophaceous gout
2) Eruptive xanthomata
3) Condyloma accuminata
4) Adenoma sebaceum
5) Chickenpox
Typical lesions of eruptive xanthomata are shown: raised lesions, typically
occurring on extensor surfaces. Eruptive xanthomata occur in
hyperlipidaemic states associated with hypertriglyceridaemia. Well-
recognised associations of hypertriglyceridaemia include acute
pancreatitis and lipaemia retinalis.
What is the diagnosis?
1) Anterior uveitis
2) Dendritic ulcer
3) Rubeosis iridis
4) Episcleritis
5) Scleromalacia perforans
Dendritic ulcers are caused by Herpes simplex virus. Presentation is
usually with pain, photophobia, blurred vision , conjunctivitis and
chemosis. Steroid eye drops are contraindicated as they may induce
massive amoeiboid ulceration and blindness. Diagnosis is by instillation of
fluorescein eye drops which stain the ulcer (shown in slide).
What is the most likely diagnosis in this 22
year old woman with a history of chronic
sinusitis?
1) Brain abscess
2) Cerebral toxoplasmosis
3) Glioblastoma multiforme
4) Metastatic carcinoma
5) Histoplasmosis
Cerebral metastases, toxoplasmosis, histoplasmosis and some primary
brain tumours can appear as ring-enhancing lesions on a CT brain. On
balance, the most likely diagnosis with a slitary frontal lesion in a young
patient with chronic sinusitis is a frontal brain abscess.
A 35 year-old man with HIV disease presents
with sudden loss of vision in his right eye. He is
not curently taking antiretroviral therapy. A
recent CD4 count was measured at 20 cells/mm3.
What is the most appropriate therapy to
initiate?
1) High-dose intravenous aciclovir
2) Intravenous ganciclovir
3) Zidovudine + lamivudine + nevirapine
4) Sulfadiazine + pyrimethamine
5) Amphotericin B + flucytosine
The slide shows the typical 'cottage cheese and tomato ketchup' or 'pizza' appearance of CMV
retinitis in a patient with HIV disease. Intravenous ganciclovir is currently the treatment of
choice. Available forms of ganciclovir include intravenous and oral preparations as well as an
ocular implant. The principal disadvantages of the latter two are: the poor bioavailability of the
oral preparation; the implant is effective at clearing ocular disease, but CMV is disseminated at
this stage of AIDS and the implant does not address systemic infection. Pyrimethamine +
sulfadiazine are used to treat toxoplams retinitis; zidovudine + lamivudine + nelfinavir
constitutes highly active antiretrovral ('combination') therapy (HAART) for HIV disease.
Initiation of HAART has been shown to worsen active CMV eye disease and should be witheld
until CMV is in remission.
This 80 year-old lady presented with congestive
cardiac failure.
What is the diagnosis?
1) Scurvy
2) Ricketts
3) Paget's disease of bone
4) Syphilis
5) Achondroplasia
Typical appearance of Pagetic tibiae is shown, with characteristic bowing.
Complications of Paget's disease include: bone pain, pathological fractures,
nerve deafness, spinal cord compression, high-output cardiac failure and
osteosarcoma. The latter affects ~10% of patients who have had the disease for
over 10 years. Bowing of the tibia may also be a feature of ricketts, syphilis and
achondroplasia.
This 34 year-old gay man was referred to the
hospital by his dentist with a painful oral
lesion.
What is the infectious agent that caused this
appearance?
1) Candida albicans
2) Human herpesvirus-8
3) Epstein-Barr virus
4) Herpes simplex virus
5) Treponema pallidum
The picture shows characteristic purple lesions of Kaposi's sarcoma (KS) on the hard
palate; it is seen almost exclusively in gay men with HIV disease. In HIV infection, KS
lesions occur most commonly on the face; however lesions may be widely disseminated in
skin, bronchial tree and G-I tract. KS can occur at any time during the course of HIV
disease, irrespective of CD4 count; disseminated disease is commoner in late stage HIV
disease/AIDS. In all cases, KS is caused by human herpesvirus 8 infection (HHV-8),
probably sexually acquired.
This patient has an abnormal chest x-ray.
What is the most likely diagnosis?
1) Amyloidosis
2) Sarcoidosis
3) Secondary syphilis
4) Squamous cell carcinoma
5) Tuberculosis
The slide shows cutaneous tuberculosis (lupus vulgaris). Cutaneous TB
usually occurs due to spread from an endogenous source; >80% of cases
occur on the face and neck. Lesions begin as papules and coalesce to form a
plaque. The centre of the lesion consists of scar tissue while the lesion
extends from the periphery; apple-jelly nodules are classically described at
the margins of the lesions.
What is the diagnosis?
1) Rheumatoid arthritis
2) Osteoarthritis
3) Tophaceous gout
4) Sarcoidosis
5) Psoriatic arthropathy
The slide shows the typical appearance of chronic tophaceous gout.
Immunofluorescent staining of a skin
biopsy from this patient showed
intercellular deposition of IgG at the
dermal-epidermal junction.
What is the diagnosis?
1) Staphylococcal scalded skin syndrome
2) Necrolytic migratory erythema
3) Pemphigoid
4) Dermatitis herpetiformis
5) Pemphigus vulgaris
Pemphigus vulgaris is a blistering disease
seen predominantly in elderly patients. It is
characterised by the formation of thin-
walled blisters that rupture easily.
Immunofluorescent staining of a biopsy
sample shows deposition of IgG on
acanthocytes at the dermal-epidermal
junction – resulting in a 'crazy paving'
effect.
A 19-year-old man presented with
pleuritic chest pain which occurred
suddenly while playing football. He
presented to the Accident & Emergency
department complaining of dyspnoea.
What is the definitive treatment for this
condition?
1) High-flow inspired oxygen
2) Low molecular weight heparin
3) Nebulized salbutamol
4) Intravenous amoxycillin +
clarithromycin
5) Intercostal chest drain insertion
The slide shows a large left sided pneumothorax with midline shift (tension
pneumothorax). An intercostal chest drain should be inserted.
Histologically, these lesions are
characterized by deposition of IgG
and C3 along the basement
membrane.
What is the diagnosis?
1) Pemphigoid
2) Staphylococcal scalded skin
syndrome
3) Mycosis fungoides
4) Pemphigus vulgaris
5) Porphyria cutanea tarda
The slide shows pemphigoid, a skin disease of the elderly (>60 years)
characterised by the development of large blisters that heal without
scarring. The condition is caused by IgG autoantibodies against
components of the basement membrane. Histology shows IgG and C3
deposition along the basement membrane.
A 32 year-old man presented to hospital with a
four week history of progressively worsening
dyspnoea on exertion. He also complained of a
non-productive cough. Over the two days
preceeding admission the patient had become
breathless at rest and was started on oral co-
amoxiclav by his general practitioner.
On examination he was febrile 38°C and looked
unwell. Candida was noted on the tonsilar pillars.
No wheeze or crackles were heard in his chest. His
chest radiograph is shown.
Which of the following is most likely to assist in
confirming the diagnosis?
1) Peak expiratory flow rate
2) Oxygen saturations pre- and post-excercise
3) Legionella urinary antigen
4) Sputum culture
5) Blood pressure measured in inspiration and
expiration
The history is characteristic of Pneumocystis carinii pneumonia (PCP). The presence of oropharyngeal candidiasis in
a patient without a known immunosuppressive illness is highly suggestive of HIV/AIDS. Salient features are: several
days/weeks of increasing dyspnoea; dry cough; marked oxygen desaturation with exercise. This can be done on an
exercise bike, or by asking the patient do do some simple exercise on the ward (sit back and forward in bed several
times; squat and stand several times; walk a flight of stairs, etc.)
What is the diagnosis?
1) Erythema annulare
2) Erythema nodosum
3) Erythema multiforme
4) Erythema marginatum
5) Erythema migrans
The slide shows typical target lesions of erythema multiforme. "EM, first
described in 1860 by von Hebra, is characterized by symmetrical target-
like lesions. These lesions are described as erythematous papules
surrounded by a raised, erythematous ring, which is in turn encircled by an
erythematous outer ring. The lesions are generally located on the
extremities"
What is the diagnosis?
1) Medullated nerve fibres
2) Central retinal vein thrombosis
3) Pre-proliferative diabetic
retinopathy
4) Papilloedema
5) Angioid streaks
The slide shows papilloedema. "Management of optic disc edema begins with a
correct diagnosis. Most importantly, it is crucial to distinguish between
papilledema and the many other forms of optic disc edema, including
'masqueraders' such as buried optic disc drusen. Consider the acuity, visual fields,
ophthalmoscopy findings and especially the laterality of presentation carefully in
the initial work-up."
A 45-year-old woman presented to
the Accident & Emergency
department with a high spiking fever
ten days after returning from holiday
in Kenya. Before leaving the U.K. her
General Practitioner had prescribed
mefloquine malaria prophylaxis. She
had started the mefloquine one week
before travelling and had continued
to take it while she was in Africa. She
complained of regular spiking fevers
and rigors and of lethargy and
malaise. Physical examination was
unremarkable apart from a
temperature of 40°C. A peripheral
blood film is shown below.
What is the diagnosis?
1) African trypanosomiasis
2) Dengue fever
3) oa loa
4) Onchocerciasis
5) Plasmodium falciparum malaria
The incubation peroid for falciparum malaria is
approximately 12 days. Chemoprophylaxis should be
started 1 week before travelling to a malaria-endemic
country and continued for 1 month after returning. Severe
malaria is defined parasitologically as >5% of rbcs
parasitised. However, the degree of parasitaemia on the
peripheral film is usually a gross underestimate of the true
parasite burden, as only younger ring forms are seen in the
peripheral circulation in falciparum malaria. In the slide
shown, the blood film shows ring forms within
erythrocytes; some erythrocytes contain 2-3 parasites per
cell – typical of falciparum; other forms of malaria seldom
have >1 parasite per red cell.
The pedigree chart below is
that of a family in which
there is a history of young
adults presenting with
subacute bilateral visual
loss.
What are the chances of the
individual labelled ‘A’ being
affected by the disease?
1) 0%
2) 25%
3) 50%
4) 75%
5) 100%
The clinical description is that of Leber’s
hereditary opic neuropathy, one of the first
diseases to be aetiologically linked with
inheritance of mitochndrial DNA.
Mitochodrial DNA is always inherited from
the mother and diseases therefore show a
distinctive maternal pattern of
transmission.100%
This is the coronary angiogram of a
34 year old male who presents with
chest pain of 2 hours duration. ECG
demonstrates widespread ST
segment depression throughout the
chest leads. Investigations reveal a
cholesterol concentration of 4.5
mmol/l, a triglyceride concentration
of 2.0 mmol/l, a fasting glucose of 4.2
mmol/l and an ESR of 10 mm/hr
with a CRP of 4.5 mg/l. Troponin T
concentration is elevated.
From the list, what is the most likely
diagnosis.
1) Aortic dissection
2) Giant Cell Arteritis
3) Kawasaki Disease
4) Polyarteritis Nodosa
5) Takayasu's disease
This patient presents with a premature acute
coronary syndrome. However, his lipid profile is
entirely normal as is glucose and inflammatory
parameters are normal arguing against any
inflammatory process. The coronary angiogram
reveals out-pouching (aneurysms) and would be
typical of previous Kawasaki disease. This is a
childhood febrile illness which results in
inflammation of the mucous membranes with a
desquamative skin rash. The condition is frequently
self-limiting but it is important to recognise as it
causes coronary arterial inflammation resulting in
aneurysm formation (25% of cases) which may
present much later in life. Coronary disease can be
prevented with treatment which includes NSAIDs
and gammaglobulin infusion.
The structure shown below
was identified on microscopy
of a sputum sample from a
patient who presented with
haemoptysis. He has an
abnormal chest radiograph.
What treatment should be
started?
1) Intravenous amphotericin B
2) Intravenous
amoxycillin/clavulanic acid +
clarithromycin
3) Intravenous cefotaxime
4) Intravenous vancomycin
5) Isoniazid + rifampicin +
ethambutol + pyrazinamide
The slide shows the typical morphology of
Aspergillus fumigatus. The patient will need
treatment with amphotericin.
A 9 year-old boy presents to
casualty with visual disturbance. A
visual field defect is detected on
confrontational testing.
Fundoscopy reveals the appearance
shown below.
What is the most likely cause of
this appearance?
1) Cytomegaloviru
2) Medullated nerve fibres
3) Retinoblastoma
4) Toxocara canis
5) Toxoplasma gondii
The slide shows the typical appearance of toxocara retinitis with a lesion at the
macula. In retinitis due to Toxocara canis, there is usually only a single, well-
demarcated lesion.
This 25 year old man presented with
a four day history of fever, malaise
and a pruritic rash. What is the
diagnosis?
1) Meningiococcal septicaemia
2) Disseminated gonococcal infection
3) Chickenpox
4) Guttate psoriasis
5) Scabies
A pruritic vesicular rash. Note traces of calamine lotion.
Chickenpox
A 77-year-old man presents with a headache.
Both pupils react equally to light and
accommodation.
The abnormality shown is probably caused by
1) A complete lesion of the left oculomotor
nerve nucleus in the midbrain
2) Compression of the right third nerve by a
posterior communicating artery aneurysm
3) Compression of the left third nerve by an
anterior communicating artery aneurysm
4) Compression of the fourth cranial nerve in
the cavernous sinus
5) Microinfarction of the left third nerve
The third (oculomotor) nerve nucleus complex lies in the midbrain. Motor neurones project to the ipsilateral
medial rectus, inferior rectus, and internal oblique muscles and to the contralateral superior rectus. One central
nucleus innervates levator palpebrae superioris bilaterally. Therefore a midbrain infarct that destroys the
central nucleus will result in bilateral ptosis. Damage to the oculomotor nerve during its course results in
ipsilateral ptosis; at rest, the globe is diverted downwards and laterally. The eye cannot be moved medially,
superiorly or inferiorly. The effect on the pupil is variable, depending on the location of the lesion. Compression
of the nerve (e.g. by tumour, posterior communicating or posterior cerebral artery aneurysms) results in an
acute total (painful) third nerve palsy with a dilated unreactive pupil. Pupillary dilatation occurs early when the
nerve is compressed since sympathetic nerve fibres that innervate the iris are carried on the outside of the nerve
bundle. Pupillary sparing is characteristic of third nerve lesions caused by infarction in patients >50 years of
age with diabetes or hypertension.
This 52 year old female presents with a one day
history of fever and this facial appearance. The
patient is penicillin allergic having had a problems
with facial swelling previously.
Which of the following antibiotics would be the
most appropriate treatment for this patient?
1) Cefuroxime
2) Chloramphenicol
3) Co-trimoxazole
4) Erythromycin
5) Vancomycin
Erysipelas is a Streptococcus pyogenes infection of the deep dermis and subcutis.
Treatment relies upon IV antibiotics such as Benzylpenicillin and Erythromycin. In a
penicillin allergic patient clearly a macrolide is the drug of choice from the above list.
There is a 10% cross allergy between cephalosporins and penicillins. Although the
organism is likely to be sensitive to both vancomycin and chloramphenicol, the latter
is used with limitations due to the incidence of aplastic anaemia and the former tends
to have restricted use due to nephrotoxicity. Complications include sepsis, cerebral
abscess and venous sinus thrombosis.
What is the diagnosis?
1) Dermatomyositis
2) Systemic sclerosis
3) Peutz-Jegher's syndrome
4) Mixed mitral valve disease
5) Facial lipodystrophy
Systemic sclerosis is a multisystem disease of unknown aetiology that is
characterised by multi-organ fibrosis. The disease affects blood vessels, skin, G-I
tract, heart, kidneys and lungs. The CREST syndrome (calcinosis, Raynaud's
phenomenon, esophageal dysmotility, sclerodactyly, telangiectasia) is a localised
form of the disease generally associated with a better prognosis. The distinction
between CREST and SSc cannot be made on the basis of a picture alone.
The plain x-ray of this man's
hands does not show any
evidence of bone erosion.
Which of the following diseases
is most likely to cause this
appearance?
1) Parkinson's disease
2) Rheumatoid arthritis
3) Gouty arthropathy
4) Osteoarthritis
5) Reiter's syndrome
The diagnosis is Jaccoud's arthropathy. The picture shows joint subluxations and
swan neck deformities, caused by recurrent episodes of synovitis that damage
tendon sheaths and slings resulting in joint deformity. Jaccoud's arthropathy is
seen in: SLE; rheumatic fever, Parkinson's disease; hypocomplementaemic
urticarial vasculitis.
A 73-year-old lady presents with anaemia and the
following blood picture.
Hb 8.5 g/dL
MCV 69 fL
WCC 5.2 x109/L
Plt 240 x109/L
Physical examination reveals the abnormality shown in the
picture. OGD and colonoscopy show no other type of
lesion.
She was transfused with 4 units of packed red cells. What
is the best course of subsequent management?
1) Argon beam ablation therapy
2) Genetic counselling
3) Nasal skin graft
4) Mefanamic acid
5) Oral iron therapy
The slide shows the typical appearance of Hereditary Haemorrhagic Telangiectasia - an autosomal dominant
condition associated with bleeding (usually nose and GI tract) from early teens and worsening after the age of
50. There may also be AV malformations of the brain, lung and GI tract.
Argon ablation is used for Gastric antral vascular ectasia (GAVE) (Gastroenterology 1984;87:1165-70). Nasal
skin grafts are used for persistent epistaxis but this lady has telangiectasia in many other places. Oestrogen
therapy can be effective especially in women but the evidence is not strong. Lancet (1990;335:953-5)
Oral iron therapy
This patient presented following a
collapse at home. He was noted to be in
first degree heart block on admission to
Casualty. What is the diagnosis?
1) Acromegaly
2) Hypopituitarism
3) Myasthenia gravis
4) Myotonic dystrophy
5) Parkinson's disease
Myotonic dystrophy is the commonest adult muscular dysrophy. Features include
frontal baldness in men, atrophy of temporalis, masseters and facial muscle. Neck
muscles (including sternocleidomastoid) are involved early in the course of
disease. Cardiac abnormalities are common, and include first degree heart block
and complete heart block.
This blood film was obtained on a 37
year old male. What is the likely cause
for these abnormalities?
1) Disseminated Intravascular
Coagulation
2) Hereditary spherocytosis
3) Malaria
4) Splenectomy
5) Trypanosmiasis
The image shows Howell-Jolly bodies. These are spherical bluish
inclusions within erythrocytes and are nuclear fragments of condensed
DNA which are normally removed by the spleen. They are seen in
severe haemolytic anaemias or in dysfunctional/asplenic patients.
A 16 year-old boy is admitted to hospital for
investigation of a chronic cough. He has a
history of recurrent episodes of bronchitis
since childhood and missed a significant
amount of school. He also gives a
longstanding history of loose stools – which
he describes as being 'greasy'. He lives with
his parents on a dairy farm. He has two
brothers who are both well. On examination
he appears small for his age and weighs
50kg. Loss of the nail angle is noted in all
digits. There are widespread coarse crackles
audible in both lung fields, most notable at
the bases. A sputum sample is sent for
culture: the culture plate is shown.
What is the most likley diagnosis?
1) Chronic granulomatous disease
2) Cystic fibrosis
3) Hypogammaglobulinaemia
4) Tuberculosis
5) Extrinsic allergic alveolitis
The history is most suggestive of CF
(chronic cough, steatorrhoea due to
pancreatic exocrine insufficiency).
The culture plate shows a growth of
Pseudomonas aeruginosa,
characterized by the green
colouration of the colonies - due to
production of the pigment
pyocyanin.
Which of the following statements regarding this disease is correct?
1) Immonufluorescent staining of a biopsy specimen will show IgG and C3 deposition
along the basement membrane
2) This appearance is due to a cutaneous T-cell lymphoma
3) Faecal porphyrins will be elevated
4) Mucosal involvement is rare
5) Systemic corticosteroids are the mainstay of treatment
The slide shows the characteristic
rash of pemphigus vulgaris. Blisters
are thin-walled and rupture easily
(intact blisters are rarely seen).
Large surface area of the body can
be affected and the mortality
without treatment is high. Mucosal
involvement is common at
presentation (unlike pemphigoid).
Treatment is with high-dose
corticosteroids.
This thoracic CT scan was
performed on Brazilian a man
who presented with dysphagia.
What is the most likely
causative organism?
1) Treponema pallidum
2) Trypanosoma cruzi
3) Mycobacterium tuberculosis
4) Leishmania donovani
5) Tropheryma whipelii
The slide shows a chagasic megaoesophagus: a massively dilated oesophagus
containing both food residue and radio-opaque contrast medium. Chagas disease is
cause by T. cruzi, which infects humans via a bloodsucking insect vector (triatomine
bugs).
A 21-year-old woman presented with a four-week
history of increasing exertional dyspnoea and a dry
cough. Her chest was clear to auscultation and she was
afebrile. Her chest radiograph is shown below.
Investigations:
Hb 13.2g/dl
WBC 3.9x109/l
Neutrophils 2.5x109/l
Lymphocytes 1.0x109/l
Monocytes 0.4x109/l
Platelets 390x109/l
Sodium 141mmol/l
Potassium 4.7mmol/l
Urea 5.5mmol/l
Creatinine 102mmol/l
Calcium 2.6mmol/l
Bilirubin 8µmol/l
Alkaline phosphatase 110µmol/l
AST 22iu/l
Total protein 55g/dl
Globulin 31g/dl
What is the most likely diagnosis?
1) Chlamydia pneumoniae pneumonia
2) Lymphoma
3) Pneumocystis carinii pneumonia
4) Sarcoidosis
5) Wegener’s granulomatosis
click
The chest x-ray shows bilateral reticulonodular shadowing and bilateral hilar
lymphadenopathy. There is lymphopoenia, hyperglobulinaemia and
hypercalcaemia.
Calculation of corrected calcium:
Add 0.1 mmol/l of calcium for every 4g/dl that the albumin level is below 40
g/dl
Albumin + globulin = Total protein
Albumin = Total protein – globulin
= 55 - 31
= 24
Corrected calcium = 2.6 + (((40-24)/4) x 0.1)
= 2.6 + ((16/4) x 0.1)
= 2.6 + (4 x 0.1)
= 2.6 + 0.4
= 3.0 mmol/l
What is the diagnosis?
1) Guttate psoriasis
2) Scabies
3) Stevens-Johnson syndrome
4) Chickenpox
5) Eosinophilic folliculitis
The slide shows chickenpox in an adult. Chickenpox is caused by varicella
zoster virus. Infection most commonly affects children aged 5-9 years. Patienst
are infectious for ~48 hours prior to the appearance of the rash and until all
lesions have crusted. Presents as a low-grade fever and malaise for 3-5 days.
Skin lesions (macules, papules and vesicles) vary in stages of evolution with
successive crops over 2-4 days.
Which of the following is not a recognized
complication of this condition?
1) Marked joint laxity
2) Acute pancreatitis
3) Angioid streaks of the fundus
4) Accelerated atherosclerosis
5) Gastrointestinal haemorrhage
The slide shows the typical 'plucked chicken
skin' appearance of pseudoxanthoma
elasticum.Marked joint laxity is there
This gentleman first presented in his early 20's
with haemoptysis. He has required intermittent
blood transfusions throughout his life.
What is the diagnosis?
1) Hereditary haemorrhagic telangiectasia
2) Wegener's granulomatosis
3) Kaposi's sarcoma
4) Hepatitis C
5) Peutz-Jegher's syndrome
Typical telangiectasia of hereditary haemorrhagic telangiectasia (also known as
Osler-Weber-Rendu syndrome) are seen. The condition is inherited as an
autosomal dominant trait; sporadic cases also occur. Patients typically present
early in adult life with anaemia due to occult bleeding from gastrointestinal
telangiectasiae. Lesions commonly occur in the mouth and on the face.
Complications of the disease include haemoptysis and pulmonary haemorrhage.
This 42 year-old lady presented with a
grand mal seizure. CT scan of her
head showed several small areas of
periventricular calcification.
What is the diagnosis?
1) Neurofibromatosis
2) Tuberose sclerosis
3) von Hippel-Lindau syndrome
4) Multiple endocrine neoplasia type
IIb
5) Gardner's syndrome
The picture shows subungual fibromas, characteristic of tuberose sclerosis. Fibromas
may also develop within the CNS, where they calcify. Other manifestations of
tuberose sclerosis include Ash-leaf macules (discrete areas of hypopigmentation; often
present at birth), shagreen patch (rough patch of skin with a texture similar to shark
skin), adenoma sebaceum (angiofibromas of the face).
This HIV positive man presented
with cough, haemoptysis and a
rash (shown).
What is the causative agent?
1) Mycobacterium tuberculosis
2) Pneumocystis carinii
3) Cytomegalovirus
4) Epstein-Barr virus
5) Human herpesvirus-8
The picture shows characteristic purple lesions of Kaposi's sarcoma (KS); it is seen
almost exclusively in gay men with HIV disease. In HIV infection, KS lesions occur
most commonly on the face; however lesions may be widely disseminated in skin,
bronchial tree and G-I tract. Pulmonary disease can present with dyspnoea and
cough with haemoptyis; it is usually associated with cutaneous disease, but can
present in isolation. Chest x-ray may show pulmonary nodules. KS is caused by
human herpesvirus-8 (HHV-8) infection.
This 52 year old female presents with a two
day history of fever and painful facial rash.
What is the likely diagnosis?
1) Erysipelas
2) Impetigo
3) Lupus pernio
4) Lupus Vulgaris
5) Rosacea
Erysipelas is a streptococcal pyogenes infection of the deep dermis and subcutis.
Treatment relies upon IV antibiotics such as Benzylpenicillin and Erythromycin.
Complications include sepsis, cerebral abscess and venous sinus thrombosis.
Which of the following statements are correct
regarding this patient's condition?
1) It occurs more commonly in men
2) Rheumatoid factor is positive in >90% of cases
3) It is associated with an erosive arthritis
4) Raynaud's phenomenon is a feature in ~10%
5) It is associated with a reduced transfer factor
The slide shows a patient with systemic sclerosis (SSc). Features present include
microstomia, facial telangiectasia and beaking of the nose. Rheumatoid factor is
positive in 25% of cases; positive ANF in 95%. Autoantibodies strongly associated
with SSc include antinucleolar antibodies and anti-U3 nucleolar RNP.
Anticentromere antibodies are strongly associated with CREST syndrome. The
CREST syndrome (calcinosis, Raynaud's phenomenon, esophageal dysmotility,
sclerodactyly, telangiectasia) is a localised form of SSc generally associated with a
better prognosis. The distinction between CREST and SSc cannot be made on the
basis of a picture alone. It is associated with a reduced transfer factor
You are called to see a 58-year-old lady
on the ward because the nursing staff
have noticed the following abnormality of
her eye. She is an inpatient with a history
of paroxysmal atrial fibrillation and
rheumatoid arthritis. The lady complains
only of a minor ache in the eye.
Which of her medication is a likely cause
of the lesion?
1) Amiodarone
2) Aspirin
3) Folic acid
4) Meloxicam
5) Methotrexate
The picture shows a subconjunctival haemorrhage which is an alarming adverse
effect of aspirin therapy (and other antiplatelets). It usually resolves over 10 - 14
days. If the haematoma is large it may be worth considering prophylactic antibiotic
eyedrops.
The genetic tree shown is that of a
family in which there is a history of
cystic fibrosis.
Assuming that the gene frequency in
the general population is 1/20, what is
the chance of individual ‘A’ having
the disease?
1) 1/20
2) 1/80
3) 1/120
4) 1/160
5) 1/320
Cystic fibrosis is inherited as an autosomal recessive disease. In this case, since both parents are
unaffected they must both be carriers in order to have a child with CF. The chances of the mother
being a carrier is 2/3 (the mother's parents must both be carriers since she has a brother and a sister
with the disease, and since she is unaffected she must either be a carrier or 'normal'). We must
assume that the chance of the child's father being a carrier is 1/20 (the gene frequency in the general
population); and the chances of two carriers of the CF gene having a child with the disease is 1/4.
Therefore, the chance of individual 'A' being affected is: 2/3 x 1/4 x 1/20 = 1/120
This 72 year-old lady presented
with dysphagia.
Which of the following
autoantibodies is highly specific
for this condition?
1) Rheumatoid factor
2) Anticardiolipin antibody
3) Antinuclear factor
4) Anticentromere antibody
5) Antimitochondrial antibody
The slide shows the hands of a patient with systemic sclerosis/CREST (the distinction
cannot be made on the basis of the picture alone). Rheumatoid factor is non-specific and is
positive in 25% of cases; ANF is positive 95%. Autoantibodies strongly associated with SSc
include antinucleolar antibodies and anti-U3 nucleolar RNP. Anticentromere antibodies are
strongly associated with CREST syndrome. Anti-SSA (Ro) and anti-SSB (La) antibodies are
seen in SSc associated with Sjogren’s syndrome. Anticardiolipin antibodies may be found in
association with SLE. Antimitochondrial antibodies are associated with primary biliary
cholangitis. Anti-RNP antibodies are also associated with mixed connective tissue disease.
This patient was brought in to
hospital unresponsive.
Which of the following is the
most appropriate initial step
in management?
1) CT head scan
2) Blood cultures
3) Full blood count
4) Lumbar puncture
5) Administration of
intravenous cefotaxime
The picture shows a purpuric rash of meningiococcal septicaemia. All options are
relevant in the overall management of a patient with meningiococcal disease.
However, administration of iv antibiotics is the greatest priority – regardless of
whether cultures have been sent.
An 82-year-old man presents with weight loss
(5kg) and a hoarse voice of two months’
duration. His chest radiograph is shown below.
What clinical signs are likely to be found?
1) Dilated left pupil
2) Left ptosis
3) Inability to blink left eyelid
4) Inability to sweat on the right upper body
5) Right-sided exophthalmos
The chest x-ray shows left upper lobar consolidation; the history of hoarseness implies
involvement of the recurrent laryngeal nerve – most likely due to invasion by tumour. Upper
lobar malignancies involving the superior pulmonary sulcus can destroy surrounding structures
leading to a characteristic clinical pattern – Pancoast's syndrome. The syndrome consists of pain
in a C8-T2 distribution (caused by infiltration of these nerves) often accompanies by radiological
evidence of destruction of the first and second ribs. Horner's syndrome frequently co-exists due to
infiltration of the sympathetic trunk. Horner's syndrome consists of enophthalmos, ptosis, miosis
and ipsilateral loss of the ability to sweat).
This 34 year old female presents with a nine month
history of weight gain, weakness and amenorrhoea.
Examination reveals the appearances as shown, a blood
pressure of 180/110 mmHg and proximal myopathy.
Urinalysis shows ++ glucose and a pregnancy test is
negative.
Which of the following would be the most appropriate
diagnostic test?
1) Random Cortisol concentration
2) Oestradiol concentration
3) Oral glucose tolerance test
4) Pituitary MRI scan
5) Urine free cortisol estimation
This patient appears typically Cushingoid. She is also hypertensive, has glycosuria suggesting
diabetes and has a proximal myopathy. The diagnosis is likely to be Cushing's syndrome and the
most appropriate investigation would be urine free cortisol estimation. Elevated UFC has high
sensitivity and specificity (above 95%) for the diagnosis of Cushing's syndrome. Another useful
screening test would be the 1mg overnight dexamethasone suppression test which has similar
sensitivity and specificity. Features that would argue against pseudo-cushing's due to obesity or
PCOS would be proximal myopathy, easy bruising and thin skin. Amenorrhoea is pften
associated and is due to hypogonadotrophic hypogonadism.
What is the diagnsosis?
1) Tuberculoid leprosy
2) Pyoderma gangrenosum
3) Morphoea
4) Necrobiosis lipoidica dibeticorum
5) Pre-tibial myxoedema
Necrobiosis lipoidica is associated with
diabetes mellitus. It is commoner in
females than males. The lesions are
painless.
This patient presents with a sudden onset of
frontal headache, diplopia and vomiting.
Examination reveals the appearances as shown.
There is also a constriction of outer visual fields
in both eyes, a loss of sensation over the
left/right upper maxilla. Investigations reveals
a sodium of 128 mmol/l, a potassium 4.2
mmol/l, a urea of 4.2 mmol/l and a creatinine of
67 micromol/l with a plasma glucose of 3.8
mmol/l.
Which of the following diagnoses is most likely:
1) Anterior communicating artery aneurysm
rupture
2) Lateral Medullary syndrome
3) Meningitis
4) Pituitary apoplexy
5) Weber's syndrome
This patient has a sudden onset headache and features of a cavernous sinus compression but with compression
of the chiasm as suggested by the bitemporal hemianopia. This suggests pituitary apoplexy. The mild
hyponatraemia may be a consequence of the either SIADH or secondary hypoadrenalism. Pituitary apoplexy
usually occurs in a pre-existent pituitary tumour which may be entirely asymptomatic before presentation.
Weber’s syndrome is a mid brain stroke with ipsilateral third nerve palsy and contralateral hemiplegia.
Wallenberg's syndrome or lateral medullary syndrome is due to infarction of the posterior inferior cerebellar
artery resulting in ipsilateral V, IX, X nerve involvement, dissociated sensory loss and ataxia.
A 68 year-old man presented to
casualty complaining of high
fevers and night sweats two
weeks after discharge from
hospital following a left knee
replacement. Of note, he had
undergone splenectomy 20 years
previously after he suffered
splenic rupture in a road traffic
accident. On examination, his left
knee appeared swollen and was
hot to touch. Aspirate of the joint
was sent for microscopy. Gram's
stain of the aspirate is shown
below.
What is the causative organism?
1) Candida albicans
2) Enterococcus faecalis
3) Escherichia coli
4) Staphylococcus aureus
5) Streptococcus pyogenes
The Gram stain shows Gram positive cocci growing in
clusters – typical of Staphylococcus aureus. This is the most
likely organism to cause post-operative infection of prosthetic
joints within the first 1-4 weeks following surgery.
This 34 year old female presents with a one
month history of weight loss, fatigue and this
painless rash on her shin.
Which of the following investigations would
you choose for this patient?
1) ACE concentrations
2) Anti-TSH receptor antibodies
3) Chest X-ray
4) Fasting plasma glucose
5) OGD with duodenal biopsy
This is pre-tibial myxoedema with slightly raised, pinkish, indurated patches
usually on the fronts of the shins or dorsum of the foot and often associated with
acropachy. A high titre of TSH receptor antibody is invariably associated with the
dermopathy.
This 42 year-old man was referred to hospital
by his dentist.
What is the diagnosis?
1) Addison's disease
2) Kaposi's sarcoma
3) Malignant melanoma
4) Mucor mycosis
5) Wegener's granulomatosis
The picture shows characteristic purple lesions of Kaposi's sarcoma (KS) on the hard
palate. The disease was originally described by Kaposi as a rare tumour in elderly men of
Mediterranean origin ('classical Kaposi's sarcoma') in whom KS was usually found on the
lower legs and feet. KS is now most commonly associated with HIV disease ('epidemic
Kaposi's sarcoma'); it is seen predominantly in gay men. In HIV infection, KS lesions
occur most commonly on the face; however lesions may be widely disseminated in skin,
bronchial tree and G-I tract. KS can occur at any time during the course of HIV disease,
irrespective of CD4 count; disseminated disease is commoner in late stage HIV
disease/AIDS.
This 22 year-old man presented to hospital with sudden
opnset of dyspnoea, cough and haemoptysis. He was
recently diagnosed HIV positive with a CD4 count of
580 cells/mm3. His chest x-ray showed a nodular
infiltrate.
What is the most likely diagnosis?
1) Cryptococcosis
2) Pneumocystis carinii pneumonia
3) Cytomegalovirus pneumonia
4) Kaposi's sarcoma
5) Varicella pneumonia
The slide shows the typical rash of chicken pox (residue of calamine lotion can be seen on his
lower back). Varicella pneumonia occurs in up to 20% of adults with chickenpox, appearing 3-5
days into the course of the illness. Symptoms include tachypnoea, cough, dyspnoea, and fever.
Cyanosis, pleuritic chest pain and haemoptysis are common. CXR may show parenchymal
nodules. The other HIV-related opportunistic infections listed, with the exception of KS, do not
present with a rash and are associated with profound immunosuppression (CD4 < 200 cells/mm3
for PCP; CD4 < 50 cells/mm3 for the others). KS can present at any stage of disease.
This 55 year old male is referred with a
long history of infrequent shaving and
low libido. Examination reveals the
appearances as shown. Investigations
reveal a testosterone of 6 nmol/l (NR 9-24
nmol/l), a LH of 22.1 (NR 3 - 10) and FSH
of 28 (NR 3 - 10).
Which of the following is the most
appropriate investigation
1) Karyotype
2) MRI pituitary
3) Prolactin concentration
4) Short synacthen test
5) Thyroid function tests
This patient has a hypogonadal appearance with gynaecomastia and the results demonstrate
primary hypogonadism. This suggests Klinefelter's and karyotype would be the most
appropriate investigation. Hyperprolactinaemia would cause Hypogonadotrophic
hypogonadism. An MRI of the pituitary would be unhelpful as the pituitary is functioning
normally.
A 35 year-old woman with sickle cell
disease was admitted to hospital
with abdominal pain. This
appearance was noted on
fundoscopy.
What is the diagnosis?
1) Branch retinal artery occlusion
2) Angioid streaks
3) Papilloedema
4) Central retinal vein occlusion
5) Lipaemia reinalis
The slide shows angioid streaks, caused by breaks in Bruch's membrane. Angioid
streaks may be associated with pseudoxanthoma elasticum, Ehlers-Danlos
syndrome, sickle cell disease and Paget's disease.
A 25-year-old man presents with vague chest pain and cough. His Chest X-ray, taken in
the Emergency Department, is shown.
What is the most appropriate treatment for this condition?
1) Amoxycillin/clavulanate + clarithromycin
2) Nebulized salbutamol
3) High-flow inspired oxygen
4) Intercostal chest drain insertion
5) Low molecular weight heparin
The slide shows a small right
apical pneumothorax. High-flow
inspired oxygen is sufficient
treatment; neither aspiration
nor chest drain insertion is
required.
This 55-year-old man presented
with a painful, swollen finger and a
fever (37.7°C).
What is likely to be found on
microscopy of aspirated synovial
fluid?
1) Bipyramidal crystals that exhibit
strong positive birefringence under
polarised light
2) Gram positive cocci in clusters
3) Needle-shaped crystals that
exhibit strong negative
birefringence under polarised light
4) Rhomboid crystals that exhibit
weak positive birefringence under
polarised light
5) Small, non-birefringent crystals
visible only under electron
microscopy
An acute attack of monoarticular arthritis is typical of gout.
Agonising pain is usually accompanied by joint swelling,
redness and tenderness, often with a low-grade fever. Urate
crystals are needle-shaped, and exhibit strong negative
birefringence under polarised light microscopy.
Calcium pyrophosphate crystals are described in answer (D);
calcium oxalate crystals are described in (A); calcium
hydroxyapatite crystals are described in (E). Septic arthritis
caused by Staphylococcus aureus (option (B)) is unlikely to
present as a monoarticular arthritis and would typically be
associated with a higher temperature; the disgnosis can only
be definitively excluded by joint aspiration, but the most likely
diagnosis remains gout in this scenario.
This 28 year-old man presented to hospital after becoming progressively more breathless over the
preceeding day. He had developed a dry cough and reported expectoration of bright red blood. He gave a
history of malaise and low-grade fever for 5 days. The rash (pictured) had appeared three days before
presentation.
What is the most likely diagnosis?
1) Varicella pneumonia
2) Goodpasture's syndrome
3) Meningiococcal septicaemia
4) Wegener's granulomatosis
5) Tuberculosis
The slide shows the typical
rash of chickenpox.
Varicella pneumonia
occurs in up to 20% of
adults with chickenpox,
appearing 3-5 days into
the course of the illness.
Symptoms include
tachypnoea, cough,
dyspnoea, and fever.
Cyanosis, pleuritic chest
pain and haemoptysis are
common. CXR shows
patchy shadowing. About
30 young adults die each
year from varicella
pneumonia and they are
best managed on a High-
Dependency Unit.
With what diseases is this appearance
associated?
1) Acromegaly
2) Poorly controlled diabetes mellitus
3) Pseudoxanthoma elasticum
4) Chronic lymphocytic leukaemia
5) Hypertriglyceridaemia
6) Paget's disease of bone
7) Acquired Immune Deficiency Syndrome
8) Retinitis pigmentosa
9) Cytomegalovirus infection
10) Closed angle glaucoma
The slide shows angioid streaks, caused by breaks in Bruch's membrane. Angioid streaks may
be associated with pseudoxanthoma elasticum, Ehlers-Danlos syndrome, sickle cell disease and
Paget's disease.
This lady's appearance has changed dramatically over the past week. She started a course of treatment four
days ago. Blood was drawn prior to treatment (Results 1, below) and repeated urgently today (Results 2, below).
Results 1 Results 2
T4 99 2 (NR 5-12 ug/dl)
TSH <0.04 28 (NR 0.4 – 6.0 mU/l)
What treatment has she received?
1) Propylthiouracil
2) Radioiodine
3) Carbimazole
4) Propranolol
5) Prednisolone
The slide shows malignant exophthalmos: malignant refers to the rapidity of onset and threat to
eyesight rather than association with malignancy. Radioiodine therapy can worsen thyroid-
associated ophthalmopathy – patients with thyroid eye disease are generally treated with steroids
for 1-2 weeks prior to starting radioiodine therapy. Treatment for malignant exophthalmos is
rapid administration of steroids. Where sight is threatened, orbital decompression may be
necessary.
The following 12-lead ECG
was obtained from a 48-year-
old man following successful
resuscitation from a
community cardiac arrest. He
had an extensive list of current
and previous medication in his
wallet.
Which two of the drugs may
have been the cause?
1) Amiloride
2) Amiodarone
3) Digoxin
4) Fentanyl patch
5) Haloperidol
6) Lignocaine
7) Ramipril
8) Salbutamol
9) Spironolactone
10) Temazepam
A large number of drugs have been implicated in causing long QT
interval and Torsade-de-pointes ventricular tachycardia
(http://www.torsades.org/druglist.cfm). Digoxin causes a shortened
QT interval. Amiloride, Ramipril and Spironolactone could lead to
hyperkalaemia but it is hypokalaemia that is associated with Torsade-
de-pointes VT. Lignocaine (and other Class 1b antiarrhythmics)
shortens phase 2 of the cardiac cycle and therefore does not prolong
the QT interval. Temazepam and Fentanyl have no effect on the QT
interval.
What organism has been cultured
from a sputum sample from a 15
year old girl with a chronic cough
and diarrhoea?
1) Pseudomonas aeruginosa
2) Staphylococcus aureus
3) Klebsiella pneumoniae
4) Haemophilus influenzae
5) Mycobacterium tuberculosis
The culture plate shows a growth of Pseudomonas aeruginosa, characterized by the
green colouration of the colonies - due to production of the pigment pyocyanin. The
history is consistent with a diagnosis of cystic fibrosis: bronchiectasis associated with
CF frequently results in recurrent infections with Pseudomonas.
This 36-year-old lady was found to have hepatosplenomegaly.
Which of the following are recognised complications of her disease?
1) Acute pancreatitis
2) Ischaemic heart disease
3) Polyarticular arthritis
4) Pathological bone fractures
5) Tetany
The slide shows yellow papules (pingeculae) in the cornea; these are characteristic of
Gaucher disease. Gaucher disease is inherited as an autosomal recessive disease. Disease is
caused by a deficiency of the enzyme glucocerbrosidase, essentail for the metabolism of
glycolipids. There are three types of Gaucher disease:
(1) Type 1 (chronic non-neuropathic; adult Gaucher disease)
(2) Type 2 (acute neuropathic; infentile Gaucher disease)
(3) Type 3 (subacute neuropathic; juvenile Gaucher disease)
Patients with all types of disease have hepatosplenomegaly and large glucocerebroside-rich
cells (Gaucher cells) infiltrating the bone marrow. Type 2 (infentile Gaucher disease) carries
the worst prognosis, with children seldom surviving beyond 2 years. Type 1 disease is the
commonest, usually presenting in childhood with hepatosplenomegaly, but not uncommonly
in middle- or old-age. Bone marrow replacement and hypersplenism result in anaemia and
thrombocytopoenia. Pathological bone fractures and avascular necrosis of the femoral heads
are not uncommon. Bony disease may be confined to the distal ends of the femurs, with
formation of characteristic ‘Erlenmeyer flask’ shaped cysts. The skin may show a grey-
brown discolouration, especially around the forehead, hands and pre-tibial regions.
Characteristic yellow or yellow-brown papules (pingeculae) develop at the sclerocorneal
junctions.
Hyperlipidaemias manifest in the eye as corneal arcus senilis; hypertriglyceridaemia is
associated with acute pancreatitis. Chronic hypercalcaemia can result in calcium deposition
in the iris; hypercalcaemia is associated with tetany. Alkaptonuria usually presents in late-
or middle-age with degenerative joint disease – ochronotic arthritis may resemble
rheumatoid disease.
This 45 year-old white female
presented to her General Practitioner
asking for advice regarding removal
of these lesions for comsetic reasons.
What additional investigation should
be performed?
1) Biopsy of lesion for histology
2) Serum Cryptococcal antigen assay
3) HIV antibody test
4) Tzanck smear
5) Herpes virus serology
Umbilicated, pearly papules 2-5mm in diameter. Seen in patients with advanced
HIV/AIDS (CD4 count <200 cells/mm3). Caused by a pox virus. Commonly occur on
face, especially near the eyelids; also occur on genitals and trunk. Treated with
cryotherapy, liquid nitrogen or curettage.
What is the diagnosis?
1) Addison's disease
2) Oral candidiasis
3) Lichen planus
4) Leukoplakia
5) Systemic sclerosis
The slide shows lichen planus on the oral
mucosa. Lesions are usually raised, flat-
topped and violaceous with a fine white
lace-like surface pattern (Wickham’s
striae). Oral lesions are common.
Which of the following is
the most important in
establishing the cause of
these lesions?
1) Anti-streptolysin O titre
2) Skin biopsy
3) Chest X-ray
4) Erythrocyte
sedimentation rate
5) Serum angiotensin
converting enzyme (ACE)
Erythema nodosum is shown in the slide. The commonest precipitant is a
streptococcal infection. However, the commonest potentially serious causes (an
therefore those that should be excluded first) include sarcoidosis and tuberculosis. A
chest X-ray is an important investigation to exclude both of these causes.
These are the hands of a 44 year old male who presents
with abdominal pain. What is the cause for his
abdominal pain?
1) Biliary colic
2) Diabetic ketoacidosis
3) Mesenteric ischaemia
4) Pancreatitis
5) Splenic infarction
This patient has a type III hyperlipidaemia which is characterised by excess
intermediate density lipoprotein (IDL) concentrations giving equally elevated total
cholesterol and triglyceride concentrations. Typically, this condition is associated with
premature atherosclerotic disease but also pancreatitis may occur.
What is the most likely cause for
this appearance?
1) Adenovirus
2) Cytomegalovirus
3) Herpes simplex virus
4) Pseudomonas aeruginosa
5) Staphylococcus aureus
Dendritic ulcers are caused by Herpes simplex virus. Presentation is usually
with pain, photophobia, blurred vision , conjunctivitis and chemosis. Steroid eye
drops are contraindicated as they may induce massive amoeiboid ulceration and
blindness. Diagnosis is by instillation of fluorescein eye drops which stain the
ulcer (shown in slide).
What does this ECG show on a
62 year old male who attends
for a BUPA health check?
1) 1st degree heart block
2) Left bundle branch block
3) Left ventricular
hypertrophy
4) Right Bundle branch block
5) Sinus rhythm
This ECG shows a prolonged PR interval of 6 small squares 0.24s. The normal PR
interval is 0.12 – 0.2 s. Causes of 1st degree heart block include increased vagal tone
(such as in trained athletes), Ischaemic Heart Disease, rheumatic fever, hyperkalaemia,
hypokalaemia, or drug therapy such as digoxin or beta-blockers. A long PR interval on
the ECG may also be caused by structural abnormalities such as an Atrial Septal
Defect. No treatment is usually required.
The structure shown below was
identified on microscopy of a
sputum sample from a patient
who presented with
haemoptysis.
What is the diagnosis?
1) Aspergillosis
2) Staphylococcal pneumonia
3) Goodpasture’s syndrome
4) Pneumoconiosis
5) Tuberculosis
The slide shows the typical morphology of Aspergillus fumigatus.
A 42-year-old Zimbabwean man
was admitted to Casualty after
becoming unconcious at
Heathrow airport shortly after
his arrival in the UK. His wife
said that he had been unwell for
several weeks, but had
complained of a high fever
associated with neck stiffness
for the past 24 hours.
Microscopy of his CSF is
pictured.
What organism is shown?
1) Listeria monocytogenes
2) Cryptococcus neoformans
3) Pneumocystis carinii
4) Toxoplasma gondii
5) Candida albicans
Diagnosis is by demonstration of C. neoformans in the
CSF – shown on an India ink stain: the thick
polysaccharide capsule is highlighted around the cell
(shown in slide). Cryptococcal meningitis is an AIDS-
defining illness occurring when CD4< 50 cells/mm3 and
may be associated with a pneumonitis. Cryptococcus can
also cause papular skin lesions that resemble molluscum
contagiosum. The disease is commoner in African
populations.
What is the diagnosis?
1) Central retinal artery occlusion
2) Cytmegalovirus retinitis
3) Retinal detachment
4) Toxoplasma retinitis
5) Central retinal vein occlusion
The slide shows the typical appearance of central retinal vein occlusion.
"Patients usually present with painless loss of vision and are found to have diffuse
retinal hemorrhages in all four quadrants of the retina as well as dilated, tortuous
veins. cotton-wool spots, disc edema, optociliary shunt vessels and neovessels might
also be present. Multiple etiologies should be considered including: hypertension,
glaucoma, optic disc edema, hypercoagulable states, vasculitis, drug-induced, and
retrobulbar compression by tumors or grave's opthalmopathy."
This 28 year-old man presented to hospital after
becoming progressively more breathless over the
preceeding day. He had developed a dry cough and
reported expectoration of bright red blood. He gave a
history of malaise and low-grade fever for 5 days. The
rash (pictured) had appeared three days before
presentation.
What is the most appropriate treatment to start?
1) Intravenous benzyl penicillin + flucloxacillin
pyrazinamide + ethambutol
2) High-dose corticosteroids
3) Co-amoxiclav + clarithromycin
4) Rifampicin + isoniazid + pyrazinamide + ethambutol
5) Intravenous aciclovir
The slide shows the typical rash of chickenpox. Varicella pneumonia occurs in up to 20% of
adults with chickenpox, appearing 3-5 days into the course of the illness. Symptoms include
tachypnoea, cough, dyspnoea, and fever. Cyanosis, pleuritic chest pain and haemoptysis are
common. In adults with pneumonitis, treatment with aciclovir is warranted.
This 60 year-old man presented to
his General Practitioner with a
three-month history of proximal
muscle weakness. He was a smoker
of 20 cigarettes a day for 40 years.
What is the diagnosis?
1) Dermatomyositis
2) Polymyalgia rheumatica
3) Cushing's syndrome
4) Myotonic dystrophy
5) Ectopic parathyroid hormone
production
Dermatomyositis is an autoimmune disease characterized by polymyositis and a skin rash. The
classic purple (heliotrope) rash is seen on sun-exposed areas, especially the eyelids, nose,
cheeks, forehead, knees, knuckles and around the nail beds. The rash may be pruritic. In older
patients presenting for the first time with dermatomyositis an underlying malignancy should be
considered. Malignancies most commonly associated with dermatomyositis include lung, ovary,
breast and G-I.
This 78-year-old man presented with a
headache and blurred vision. An MRI
angiogram showed a left posterior
communicating artery aneurysm.
Which of the following clinical signs would you
expect to elicit?
1) Both pupils will react normally to light and
accommodation
2) Left pupil will be unreactive to light or
accommodation
3) Inability to blink on the left
4) Inability to elevate the globe on the left
5) Inability to frown
6) Inability to move left globe laterally
7) Left pupil will react to light but not to
accommodation
8) Left-sided facial weakness
9) Loss of sensation over the left side of the face
10) Right-sided facial weakness
The third (oculomotor) nerve nucleus complex lies in the midbrain. Motor neurones
project to the ipsilateral medial rectus, inferior rectus, and internal oblique muscles
and to the contralateral superior rectus. One central nucleus innervates levator
palpebrae superioris bilaterally. Therefore a midbrain infarct that destroys the
central nucleus will result in bilateral ptosis. Damage to the oculomotor nerve during
its course results in ipsilateral ptosis; at rest, the globe is diverted downwards and
laterally. The eye cannot be moved medially, superiorly or inferiorly. The effect on
the pupil is variable, depending on the location of the lesion. Compression of the
nerve (e.g. by tumour, posterior communicating or posterior cerebral artery
aneurysms) results in an acute total (painful) third nerve palsy with a dilated
unreactive pupil. Pupillary dilatation occurs early when the nerve is compressed
since sympathetic nerve fibres that innervate the iris are carried on the outside of the
nerve bundle. Pupillary sparing is characteristic of third nerve lesions caused by
infarction in patients >50 years of age with diabetes or hypertension.
What is the diagnosis in this
patient with recurrent anaemia?
1) Ehlers-Danlos syndrome
2) Marfan syndrome
3) Pseudoxanthoma elasticum
4) Porphyria cutanea tarda
5) Tendon xanthomata
The slide shows cigarette-paper (papyraceous) scarring of the skin. Ehlers-
Danlos syndrome (EDS) is associated with joint laxity, hyperelasticity of skin,
and fragility of skin and blood vessels. The scarring, shown here, is the result if
minor trauma and subsequent slow healing. Eleven types of EDS are
recognised, based on the mode of inheritance, severity of disease, and clinical
manifestations.
This genetic tree belongs to a
family with hypophosphataemic
vitamin D-resitant rickets. The
abnormal gene is not re-
introduced into the family by
marriage or mutation.
What is the chance that
individual ‘A’ is affected?
1) 0%
2) 25%
3) 50%
4) 75%
5) 100%
Hypophosphataemic vitamin D-resitant rickets is inherited in an X-linked dominant
manner. Offspring of affected mothers, regardless of the children’s sex, have a 50%
chance of acquiring the gene.
A 24-year-old Irishman presented
with a ten-week history of
progressively worsening exertional
dyspnoea and a dry cough.
Auscultation of his chest revealed
fine inspiratory crackles to the mid-
zones. He was afebrile. His chest
radiograph is shown below. A Heaf
test was negative.
Which of the following investigations
will be most helpful in establishing a
diagnosis?
select three
9) HIV antibody test
10) Peak flow rate
11) Plasma lactate dehydrogenase
(LDH)
12) Sputum culture
13) Transbronchial lung biopsy
14) Trial of steroids
15) 24 hour urinary calcium excretion
1) Abdominal ultrasound scan
2) Atypical serology
3) Bone marrow aspiration
4) Bronchoalveolar lavage
5) CD4 T-lymphocyte count
6) Cytoplasmic antineutrophil cytoplasmic antibody (cANCA)
7) C-reactive protein (CRP)
8) Erythrocyte sedimentation rate (ESR)
The chest x-ray shows bilateral reticulonodular shadowing and bilateral hilar
lymphadenopathy. The most likely diagnosis is sarcoidosis. Most cases of
sarcoid present between 20 and 40 years of age with a slightly increased
incidence in women. Prevalence varies amongst different ethnic populations; in
Europe, sarcoid is commonest amongst caucasians – and has a significantly
higher incidence in the Irish. A definitive diagnosis can only be made with
biopsy evidence of a non-caseating granulomatous inflammatory process.
Supportive evidence of sarcoid is provided by elevated serum angiotensin
converting enzyme (ACE) levels, elevated 24-hour urine calcium excretion,
increased uptake on gallium 67 scanning. Elevated inflammatory markers
(ESR, CRP) are non-specific. Elevated serum calcium, though classical of
sarcoid, is actually relatively rare.
The main differential diagnosis in this case is HIV disease. Hilar adenopathy is
common in HIV disease and CXR abnormalities are common. PCP frequently
presents with a protracted history of worsening dyspnoea and dry cough with
reticular shadowing on the CXR. Lymphopoenia is a common feature n both
sarcoidosis and HIV. In this setting an HIV test is essential. CD4 counts cannot
be used as a surrogate test of HIV infection since CD4 counts can be within the
normal range in HIVseropositive individuals and is an ethically questionable
means of trying to establish a diagnosis for which there is a definitive test.
This 64 year old male presents with a six
month history of weight loss and
diarrhoea. Examination reveals the
appearances as shown.
Which of the following investigations
would be most appropriate in this case?
1) Anti-endomysial antibodies
2) Barium enema
3) Sigmoidoscopy
4) Stool culture
5) Urine hydroxy indoleacetic acid
This patient has a fixed facial flush and symptoms suggestive of Carcinoid syndrome. Therefore,
Urine HIAA concentrations would be elevated. Other features that would be expected in carcinoid
syndrome include hepatomegaly, right sided valvular lesions (as tumour products are metabolised
by the lungs) and rarely pellagra. None of the other investigations listed would provide any
diagnostic information.
A 55 year old vagrant was brought to the Casualty
department after being found unresponsive by a paramedic
crew. His chest x-ray is shown.
Select the three most important immediate investigations
that should be performed?
1) CT scan thorax
2) Sputum for acid fast bacilli and mycobacterial culture
3) Carbon monoxide transfer factor
4) Mantoux test
5) Ventilation/perfusion (V/Q) scan
6) HIV antibody test
7) Sputum for viral culture & isolation
8) CT scan brain
9) Plasma antineutrophil anticytoplasmic antibody (ANCA)
assay
10) Lumbar puncture
11) Peak expiratory flow rate
12) Bronchoscopy and transbronchial biopsy
13) Arterial blood gases
14) Intercostal chest drain insertion
15) Serum antibodies to atypical respiratory pathogens
MRCP 2 - Pictures.ppt for osce examination board revi3
MRCP 2 - Pictures.ppt for osce examination board revi3
MRCP 2 - Pictures.ppt for osce examination board revi3
MRCP 2 - Pictures.ppt for osce examination board revi3
MRCP 2 - Pictures.ppt for osce examination board revi3
MRCP 2 - Pictures.ppt for osce examination board revi3

More Related Content

Similar to MRCP 2 - Pictures.ppt for osce examination board revi3

Multiple endocrine neoplasia (men) syndromes
Multiple endocrine neoplasia (men) syndromesMultiple endocrine neoplasia (men) syndromes
Multiple endocrine neoplasia (men) syndromesMarwa Khalifa
 
Aipgmee2011part1
Aipgmee2011part1Aipgmee2011part1
Aipgmee2011part1isha9
 
Eosiniphil in dermatology
Eosiniphil in dermatologyEosiniphil in dermatology
Eosiniphil in dermatologychandraushavns
 
Neuroblastoma urology rotation
Neuroblastoma urology rotation Neuroblastoma urology rotation
Neuroblastoma urology rotation Stav Debi
 
DR OLATUNYA NEPHROBLASTOMA & NEUROBLASTOMA LECTURE.pptx
DR OLATUNYA  NEPHROBLASTOMA & NEUROBLASTOMA LECTURE.pptxDR OLATUNYA  NEPHROBLASTOMA & NEUROBLASTOMA LECTURE.pptx
DR OLATUNYA NEPHROBLASTOMA & NEUROBLASTOMA LECTURE.pptxFeniksRetails
 
NEPHROTIC SYNDROME IN PAEDIATRIC
NEPHROTIC SYNDROME IN PAEDIATRICNEPHROTIC SYNDROME IN PAEDIATRIC
NEPHROTIC SYNDROME IN PAEDIATRICMona Mofti
 
Rheumatoid Arthritis for Medical and Pharmacy Students
Rheumatoid Arthritis for Medical and Pharmacy StudentsRheumatoid Arthritis for Medical and Pharmacy Students
Rheumatoid Arthritis for Medical and Pharmacy Studentsarun chand roby
 
Rheumatology MCQs Practice questions with explanation
Rheumatology MCQs Practice questions with explanationRheumatology MCQs Practice questions with explanation
Rheumatology MCQs Practice questions with explanationDr. Almas A
 
Thyroid disases final.pdf111
Thyroid disases final.pdf111Thyroid disases final.pdf111
Thyroid disases final.pdf111alaaag
 
Disorders of growth. General Pathology
Disorders of growth. General PathologyDisorders of growth. General Pathology
Disorders of growth. General PathologyBasant AbuZaid
 
Myasthenia gravis.
Myasthenia gravis.Myasthenia gravis.
Myasthenia gravis.MohamadAlhes
 
Muscle weakness & rash (Dermatomyositis)
Muscle weakness & rash (Dermatomyositis)Muscle weakness & rash (Dermatomyositis)
Muscle weakness & rash (Dermatomyositis)Mohammad Tanvir Islam
 

Similar to MRCP 2 - Pictures.ppt for osce examination board revi3 (20)

Neuroblastoma
Neuroblastoma Neuroblastoma
Neuroblastoma
 
Multiple endocrine neoplasia (men) syndromes
Multiple endocrine neoplasia (men) syndromesMultiple endocrine neoplasia (men) syndromes
Multiple endocrine neoplasia (men) syndromes
 
Marfan syndrome for orthodontist by almuzian
Marfan syndrome for orthodontist by almuzianMarfan syndrome for orthodontist by almuzian
Marfan syndrome for orthodontist by almuzian
 
Aipgmee2011part1
Aipgmee2011part1Aipgmee2011part1
Aipgmee2011part1
 
Vasculitis
VasculitisVasculitis
Vasculitis
 
Eosiniphil in dermatology
Eosiniphil in dermatologyEosiniphil in dermatology
Eosiniphil in dermatology
 
Neuroblastoma urology rotation
Neuroblastoma urology rotation Neuroblastoma urology rotation
Neuroblastoma urology rotation
 
DR OLATUNYA NEPHROBLASTOMA & NEUROBLASTOMA LECTURE.pptx
DR OLATUNYA  NEPHROBLASTOMA & NEUROBLASTOMA LECTURE.pptxDR OLATUNYA  NEPHROBLASTOMA & NEUROBLASTOMA LECTURE.pptx
DR OLATUNYA NEPHROBLASTOMA & NEUROBLASTOMA LECTURE.pptx
 
NEPHROTIC SYNDROME IN PAEDIATRIC
NEPHROTIC SYNDROME IN PAEDIATRICNEPHROTIC SYNDROME IN PAEDIATRIC
NEPHROTIC SYNDROME IN PAEDIATRIC
 
Paraneoplastic syndromes presentation
Paraneoplastic syndromes presentation Paraneoplastic syndromes presentation
Paraneoplastic syndromes presentation
 
Rheumatoid Arthritis for Medical and Pharmacy Students
Rheumatoid Arthritis for Medical and Pharmacy StudentsRheumatoid Arthritis for Medical and Pharmacy Students
Rheumatoid Arthritis for Medical and Pharmacy Students
 
A Case of Chorea following ASV
A Case of Chorea following ASVA Case of Chorea following ASV
A Case of Chorea following ASV
 
Rheumatology MCQs Practice questions with explanation
Rheumatology MCQs Practice questions with explanationRheumatology MCQs Practice questions with explanation
Rheumatology MCQs Practice questions with explanation
 
Thyroid disases final.pdf111
Thyroid disases final.pdf111Thyroid disases final.pdf111
Thyroid disases final.pdf111
 
Brucella.pptx
Brucella.pptxBrucella.pptx
Brucella.pptx
 
Neurocutaneous syndromes
Neurocutaneous syndromesNeurocutaneous syndromes
Neurocutaneous syndromes
 
Disorders of growth. General Pathology
Disorders of growth. General PathologyDisorders of growth. General Pathology
Disorders of growth. General Pathology
 
Dermatology 5th year, 3rd lecture (Dr. Faraedon Kaftan)
Dermatology 5th year, 3rd lecture (Dr. Faraedon Kaftan)Dermatology 5th year, 3rd lecture (Dr. Faraedon Kaftan)
Dermatology 5th year, 3rd lecture (Dr. Faraedon Kaftan)
 
Myasthenia gravis.
Myasthenia gravis.Myasthenia gravis.
Myasthenia gravis.
 
Muscle weakness & rash (Dermatomyositis)
Muscle weakness & rash (Dermatomyositis)Muscle weakness & rash (Dermatomyositis)
Muscle weakness & rash (Dermatomyositis)
 

Recently uploaded

Hemodialysis: Chapter 1, Physiological Principles of Hemodialysis - Dr.Gawad
Hemodialysis: Chapter 1, Physiological Principles of Hemodialysis - Dr.GawadHemodialysis: Chapter 1, Physiological Principles of Hemodialysis - Dr.Gawad
Hemodialysis: Chapter 1, Physiological Principles of Hemodialysis - Dr.GawadNephroTube - Dr.Gawad
 
CAS 110-63-4 BDO Liquid 1,4-Butanediol 1 4 BDO Warehouse Supply For Excellent...
CAS 110-63-4 BDO Liquid 1,4-Butanediol 1 4 BDO Warehouse Supply For Excellent...CAS 110-63-4 BDO Liquid 1,4-Butanediol 1 4 BDO Warehouse Supply For Excellent...
CAS 110-63-4 BDO Liquid 1,4-Butanediol 1 4 BDO Warehouse Supply For Excellent...ocean4396
 
Connective Tissue II - Dr Muhammad Ali Rabbani - Medicose Academics
Connective Tissue II - Dr Muhammad Ali Rabbani - Medicose AcademicsConnective Tissue II - Dr Muhammad Ali Rabbani - Medicose Academics
Connective Tissue II - Dr Muhammad Ali Rabbani - Medicose AcademicsMedicoseAcademics
 
Report Back from SGO: What’s the Latest in Ovarian Cancer?
Report Back from SGO: What’s the Latest in Ovarian Cancer?Report Back from SGO: What’s the Latest in Ovarian Cancer?
Report Back from SGO: What’s the Latest in Ovarian Cancer?bkling
 
VIP Pune 7877925207 WhatsApp: Me All Time Serviℂe Available Day and Night
VIP Pune 7877925207 WhatsApp: Me All Time Serviℂe Available Day and NightVIP Pune 7877925207 WhatsApp: Me All Time Serviℂe Available Day and Night
VIP Pune 7877925207 WhatsApp: Me All Time Serviℂe Available Day and Nightpatanjali9823#S07
 
Varicose Veins Treatment Aftercare Tips by Gokuldas Hospital
Varicose Veins Treatment Aftercare Tips by Gokuldas HospitalVaricose Veins Treatment Aftercare Tips by Gokuldas Hospital
Varicose Veins Treatment Aftercare Tips by Gokuldas HospitalGokuldas Hospital
 
Gross Anatomy and Histology of Tongue by Dr. Rabia Inam Gandapore.pptx
Gross Anatomy and Histology of Tongue by Dr. Rabia Inam Gandapore.pptxGross Anatomy and Histology of Tongue by Dr. Rabia Inam Gandapore.pptx
Gross Anatomy and Histology of Tongue by Dr. Rabia Inam Gandapore.pptxDr. Rabia Inam Gandapore
 
VIII.1 Nursing Interventions to Promote Healthy Psychological responses, SELF...
VIII.1 Nursing Interventions to Promote Healthy Psychological responses, SELF...VIII.1 Nursing Interventions to Promote Healthy Psychological responses, SELF...
VIII.1 Nursing Interventions to Promote Healthy Psychological responses, SELF...JRRolfNeuqelet
 
Sell 5cladba adbb JWH-018 5FADB in stock
Sell 5cladba adbb JWH-018 5FADB in stockSell 5cladba adbb JWH-018 5FADB in stock
Sell 5cladba adbb JWH-018 5FADB in stocktammysayles9
 
VVIP Hadapsar ℂall Girls 6350482085 Scorching { Pune } Excellent Girl Serviℂe...
VVIP Hadapsar ℂall Girls 6350482085 Scorching { Pune } Excellent Girl Serviℂe...VVIP Hadapsar ℂall Girls 6350482085 Scorching { Pune } Excellent Girl Serviℂe...
VVIP Hadapsar ℂall Girls 6350482085 Scorching { Pune } Excellent Girl Serviℂe...dhyaansingh0898#S07
 
Signs It’s Time for Physiotherapy Sessions Prioritizing Wellness
Signs It’s Time for Physiotherapy Sessions Prioritizing WellnessSigns It’s Time for Physiotherapy Sessions Prioritizing Wellness
Signs It’s Time for Physiotherapy Sessions Prioritizing WellnessGokuldas Hospital
 
Tips to Choose the Best Psychiatrists in Indore
Tips to Choose the Best Psychiatrists in IndoreTips to Choose the Best Psychiatrists in Indore
Tips to Choose the Best Psychiatrists in IndoreGokuldas Hospital
 
How to buy 5cladba precursor raw 5cl-adb-a raw material
How to buy 5cladba precursor raw 5cl-adb-a raw materialHow to buy 5cladba precursor raw 5cl-adb-a raw material
How to buy 5cladba precursor raw 5cl-adb-a raw materialSherrylee83
 
Video capsule endoscopy (VCE ) in children
Video capsule endoscopy (VCE ) in childrenVideo capsule endoscopy (VCE ) in children
Video capsule endoscopy (VCE ) in childrenRaju678948
 
Sonia Journal club presentation (2).pptx
Sonia Journal club presentation (2).pptxSonia Journal club presentation (2).pptx
Sonia Journal club presentation (2).pptxpalsonia139
 
TEST BANK For Huether and McCance's Understanding Pathophysiology, Canadian 2...
TEST BANK For Huether and McCance's Understanding Pathophysiology, Canadian 2...TEST BANK For Huether and McCance's Understanding Pathophysiology, Canadian 2...
TEST BANK For Huether and McCance's Understanding Pathophysiology, Canadian 2...marcuskenyatta275
 
Bangalore whatsapp Number Just VIP Brookefield 100% Genuine at your Door Step
Bangalore whatsapp Number Just VIP Brookefield 100% Genuine at your Door StepBangalore whatsapp Number Just VIP Brookefield 100% Genuine at your Door Step
Bangalore whatsapp Number Just VIP Brookefield 100% Genuine at your Door Stepdarmandersingh4580
 
TEST BANK for The Nursing Assistant Acute, Subacute, and Long-Term Care, 6th ...
TEST BANK for The Nursing Assistant Acute, Subacute, and Long-Term Care, 6th ...TEST BANK for The Nursing Assistant Acute, Subacute, and Long-Term Care, 6th ...
TEST BANK for The Nursing Assistant Acute, Subacute, and Long-Term Care, 6th ...marcuskenyatta275
 

Recently uploaded (20)

Hemodialysis: Chapter 1, Physiological Principles of Hemodialysis - Dr.Gawad
Hemodialysis: Chapter 1, Physiological Principles of Hemodialysis - Dr.GawadHemodialysis: Chapter 1, Physiological Principles of Hemodialysis - Dr.Gawad
Hemodialysis: Chapter 1, Physiological Principles of Hemodialysis - Dr.Gawad
 
CAS 110-63-4 BDO Liquid 1,4-Butanediol 1 4 BDO Warehouse Supply For Excellent...
CAS 110-63-4 BDO Liquid 1,4-Butanediol 1 4 BDO Warehouse Supply For Excellent...CAS 110-63-4 BDO Liquid 1,4-Butanediol 1 4 BDO Warehouse Supply For Excellent...
CAS 110-63-4 BDO Liquid 1,4-Butanediol 1 4 BDO Warehouse Supply For Excellent...
 
Connective Tissue II - Dr Muhammad Ali Rabbani - Medicose Academics
Connective Tissue II - Dr Muhammad Ali Rabbani - Medicose AcademicsConnective Tissue II - Dr Muhammad Ali Rabbani - Medicose Academics
Connective Tissue II - Dr Muhammad Ali Rabbani - Medicose Academics
 
Best medicine 100% Effective&Safe Mifepristion ௵+918133066128௹Abortion pills ...
Best medicine 100% Effective&Safe Mifepristion ௵+918133066128௹Abortion pills ...Best medicine 100% Effective&Safe Mifepristion ௵+918133066128௹Abortion pills ...
Best medicine 100% Effective&Safe Mifepristion ௵+918133066128௹Abortion pills ...
 
Report Back from SGO: What’s the Latest in Ovarian Cancer?
Report Back from SGO: What’s the Latest in Ovarian Cancer?Report Back from SGO: What’s the Latest in Ovarian Cancer?
Report Back from SGO: What’s the Latest in Ovarian Cancer?
 
VIP Pune 7877925207 WhatsApp: Me All Time Serviℂe Available Day and Night
VIP Pune 7877925207 WhatsApp: Me All Time Serviℂe Available Day and NightVIP Pune 7877925207 WhatsApp: Me All Time Serviℂe Available Day and Night
VIP Pune 7877925207 WhatsApp: Me All Time Serviℂe Available Day and Night
 
Varicose Veins Treatment Aftercare Tips by Gokuldas Hospital
Varicose Veins Treatment Aftercare Tips by Gokuldas HospitalVaricose Veins Treatment Aftercare Tips by Gokuldas Hospital
Varicose Veins Treatment Aftercare Tips by Gokuldas Hospital
 
In Kuwait Abortion pills (+918133066128)@Safe abortion pills in Kuwait City
In Kuwait Abortion pills (+918133066128)@Safe abortion pills in Kuwait CityIn Kuwait Abortion pills (+918133066128)@Safe abortion pills in Kuwait City
In Kuwait Abortion pills (+918133066128)@Safe abortion pills in Kuwait City
 
Gross Anatomy and Histology of Tongue by Dr. Rabia Inam Gandapore.pptx
Gross Anatomy and Histology of Tongue by Dr. Rabia Inam Gandapore.pptxGross Anatomy and Histology of Tongue by Dr. Rabia Inam Gandapore.pptx
Gross Anatomy and Histology of Tongue by Dr. Rabia Inam Gandapore.pptx
 
VIII.1 Nursing Interventions to Promote Healthy Psychological responses, SELF...
VIII.1 Nursing Interventions to Promote Healthy Psychological responses, SELF...VIII.1 Nursing Interventions to Promote Healthy Psychological responses, SELF...
VIII.1 Nursing Interventions to Promote Healthy Psychological responses, SELF...
 
Sell 5cladba adbb JWH-018 5FADB in stock
Sell 5cladba adbb JWH-018 5FADB in stockSell 5cladba adbb JWH-018 5FADB in stock
Sell 5cladba adbb JWH-018 5FADB in stock
 
VVIP Hadapsar ℂall Girls 6350482085 Scorching { Pune } Excellent Girl Serviℂe...
VVIP Hadapsar ℂall Girls 6350482085 Scorching { Pune } Excellent Girl Serviℂe...VVIP Hadapsar ℂall Girls 6350482085 Scorching { Pune } Excellent Girl Serviℂe...
VVIP Hadapsar ℂall Girls 6350482085 Scorching { Pune } Excellent Girl Serviℂe...
 
Signs It’s Time for Physiotherapy Sessions Prioritizing Wellness
Signs It’s Time for Physiotherapy Sessions Prioritizing WellnessSigns It’s Time for Physiotherapy Sessions Prioritizing Wellness
Signs It’s Time for Physiotherapy Sessions Prioritizing Wellness
 
Tips to Choose the Best Psychiatrists in Indore
Tips to Choose the Best Psychiatrists in IndoreTips to Choose the Best Psychiatrists in Indore
Tips to Choose the Best Psychiatrists in Indore
 
How to buy 5cladba precursor raw 5cl-adb-a raw material
How to buy 5cladba precursor raw 5cl-adb-a raw materialHow to buy 5cladba precursor raw 5cl-adb-a raw material
How to buy 5cladba precursor raw 5cl-adb-a raw material
 
Video capsule endoscopy (VCE ) in children
Video capsule endoscopy (VCE ) in childrenVideo capsule endoscopy (VCE ) in children
Video capsule endoscopy (VCE ) in children
 
Sonia Journal club presentation (2).pptx
Sonia Journal club presentation (2).pptxSonia Journal club presentation (2).pptx
Sonia Journal club presentation (2).pptx
 
TEST BANK For Huether and McCance's Understanding Pathophysiology, Canadian 2...
TEST BANK For Huether and McCance's Understanding Pathophysiology, Canadian 2...TEST BANK For Huether and McCance's Understanding Pathophysiology, Canadian 2...
TEST BANK For Huether and McCance's Understanding Pathophysiology, Canadian 2...
 
Bangalore whatsapp Number Just VIP Brookefield 100% Genuine at your Door Step
Bangalore whatsapp Number Just VIP Brookefield 100% Genuine at your Door StepBangalore whatsapp Number Just VIP Brookefield 100% Genuine at your Door Step
Bangalore whatsapp Number Just VIP Brookefield 100% Genuine at your Door Step
 
TEST BANK for The Nursing Assistant Acute, Subacute, and Long-Term Care, 6th ...
TEST BANK for The Nursing Assistant Acute, Subacute, and Long-Term Care, 6th ...TEST BANK for The Nursing Assistant Acute, Subacute, and Long-Term Care, 6th ...
TEST BANK for The Nursing Assistant Acute, Subacute, and Long-Term Care, 6th ...
 

MRCP 2 - Pictures.ppt for osce examination board revi3

  • 2. What is the diagnosis? 1) Branch retinal vein occlusion 2) Papilloedema 3) Cytomegalovirus retinitis 4) Central retinal artery occlusion 5) Toxoplasma retinitis The slide shows the typical 'cottage cheese and tomato ketchup' or 'pizza' appearance of CMV retinitis in a patient with HIV disease
  • 3. This 60 year-old man presented to his General Practitioner with a three- month history of proximal muscle weakness. He was a smoker of 20 cigarettes a day for 40 years. From the following list, which would be the TWO most appropriate investigations to perform in order to establish the diagnosis? 1) Serum anti-rnp antibody 2) Serum creatine kinase 3) Skin biopsy 4) 24 hour urinary cortisol excretion 5) Muscle biopsy 6) ACTH stimulation (synachthen) test 7) Uncuffed serum calcium 8) EMG 9) Serum anti-Scl-70 antibody 10) CT scan thorax The slide shows dermatomyositis. In older patients presenting for the first time with dermatomyositis an underlying malignancy should be considered. Malignancies most commonly associated with dermatomyositis include lung, ovary, breast and G-I. The diagnosis is usually made by demonstrating elevated CK with an abnormal EMG (showing spontaneous fibrillation). Treatment is with glucocorticoids, which is often associated with remission of symptoms, even with an underlying malignancy.
  • 4. This 42-year-old lady has a history of repeated bone fractures. What is the diagnosis? 1) Familial hypercholesterolaemia 2) Gaucher disease 3) Sarcoidosis 4) Phenylketonuria 5) Wilson’s disease The slide shows yellow papules (pingeculae) in the cornea; these are characteristic of Gaucher disease. Gaucher disease is inherited as an autosomal recessive disease. Disease is caused by a deficiency of the enzyme glucocerbrosidase, essentail for the metabolism of glycolipids. There are three types of Gaucher disease: (1) Type 1 (chronic non-neuropathic; adult Gaucher disease) (2) Type 2 (acute neuropathic; infentile Gaucher disease) (3) Type 3 (subacute neuropathic; juvenile Gaucher disease) Patients with all types of disease have hepatosplenomegaly and large glucocerebroside-rich cells (Gaucher cells) infiltrating the bone marrow. Type 2 (infentile Gaucher disease) carries the worst prognosis, with children seldom surviving beyond 2 years. Type 1 disease is the commonest, usually presenting in childhood with hepatosplenomegaly, but not uncommonly in middle- or old-age. Bone marrow replacement and hypersplenism result in anaemia and thrombocytopoenia. Pathological bone fractures and avascular necrosis of the femoral heads are not uncommon. Bony disease may be confined to the distal ends of the femurs, with formation of characteristic ‘Erlenmeyer flask’ shaped cysts. The skin may show a grey-brown discolouration, especially around the forehead, hands and pre-tibial regions. Characteristic yellow or yellow-brown papules (pingeculae) develop at the sclerocorneal junctions.
  • 5. What is the diagnosis? 1)Cutaneous T-cell lymphoma 2)Metastatic malignant melanoma 3)Kaposi's sarcoma 4)Leser-Trelat sign 5)meningiococcal septicaemia The lesions shown are characteristic of Kaposi's sarcoma (KS). Lesions can vary in colour: ranging from pink-red-purple-brown; they can be macular or papular. Large lesions are often surrounded by an area of ecchymosis; they can be painful of painless.
  • 6. This 55 year old female is referred with hypertension. She has been hypertensive for over 4 years and control has been difficult despite a combination of atenolol 50 mg daily, lisinopil 20mg daily and bendrofluazide 2.5 mg daily. Examination reveals the appearances as shown with a blood pressure of 180/98 mmHg and a pulse of 66 beats per minute Which of the following investigations would be most appropriate? 1) Insulin like growth factor-1 2) Oral glucose tolerance test 3) Random Growth Hormone concentration 4) Renal ultrasound examinationrine 5) Urine catecholamine concentrations This patient appears acromegalic and hypertension may be a presenting feature. The most appropriate investigation would be an oral glucose tolerance test with GH measurements. Suppression of growth hormone concentrations below 2 mU/l would be expected in norml patients with non-suppression or paradoxical elevation seen in acromegaly. Elevated IGF-1concentrations would be expected but this is not diagnostic.
  • 7. This patient presented following a fall at home. His ECG on admission showed first degree heart block. Which of the following would be most useful in confirming the diagnosis? 1) EMG 2) Transoesophageal echocardiogram 3) Serum creatine kinase 4) Tensilon test 5) Lyme serology The slide shows typical facies of myotonic dystrophy, the commonest adult muscular dysrophy. Features include frontal baldness in men, atrophy of temporalis, masseters, facial muscle and bilateral ptosis. Neck muscles (including sternocleidomastoid) are involved early in the course of disease. Cardiac abnormalities are common, and include first degree heart block and complete heart block. Myotonic dystrophy is also associated with intellectual impairment, gonadal atrophy and insulin resistance. Diagnosis can be made on EMG and muscle biopsy.
  • 8. Which of the following agents is the most appropriate to treat this lesion? 1-Gentamicin eye drops 2-Steroid eye drops 3-Ganciclovir ocular implant 4-Aciclovir eye drops 5-Systemic glucocorticoids Dendritic ulcers are caused by herpes simplex virus. Presentation is usually with pain, photophobia, blurred vision , conjunctivitis and chemosis. Steroid eye drops are contraindicated as they may induce massive amoeiboid ulceration and blindness. Treated with aciclovir eye drops, which should be continued for three days after the ulcer has healed.
  • 9. This gentleman is being treated for ischaemic heart disease and a dilated cardiomyopathy. Which of the following agents is most likely the cause of this appearance? 1-Metoprolol 2-Frusemide 3-Isosorbide mononitrate 4-Lisinopril 5-Amiodarone The slide shows gynaecomastia. Drugs that induce gynaecomastia include: inhibitors of testosterone synthesis (ketoconazole, metronidazole, cimetidine, etomidate, cisplatin), oestrogens (digitalis, oral contraceptive pill); other drugs in which the mechanism is not known isoniazid, diazepam, omeprazole, calcium-channel blockers, ACE inhibitors, tricyclic antidepressants, busulphan, marijuanha, heroin).
  • 10. The pedigree shown relates to a disease inherited in an autosomal recessive manner. What is the chance of individual 'A' being a carrier of the abnormal gene? 1) 0% 2) 25% 3) 50% 4) 75% 5) 100% 50%The disease shows an autosomal recessive pattern of inheritance. For an affected individual to have a child with the disease, their partner must be a carrier.
  • 11. This 28 year old female presents with this appearance of her legs. What is the diagnosis? 1)Erythema ab igne 2)Erythema nodosum 3)Necrobiosis Lipoidica Diabeticorum 4)Post-phlebitic leg 5)Pre-tibial myxoedema This is pretibial myxoedema with the raised, indurated pinkish patches. Can occur anywhere but typically on the shins and dorsum of feet. Often found with acropachy and ophthalmopathy and a high titre of TSH recpetor antibodies would be expected.
  • 12. A 27 year-old man was referred to hospital with fevers and haemoptysis. Two weeks earlier he had presented to casualty following a grand mal seizure. Pending the results of sputum cultures, what is the most appropriate combination of antibiotics that should be used to treat this patient initially? 1)Amoxycillin/clavulanate + clarithromycin 2)Flucloxacillin 3)Cefuroxime + metronidazole 4)Azithromycin 5)Vancomycin + ceftazidime The slide shows an abscess in the right mid-zone. The lung abscess is likely the result of aspiration during the grand mal seizure that occurred several weeks earlier. Due to the angle of the bronchi, the right lung is more commonly affected by aspiration than the left lung. Classically, when the subject is lying down, aspirated oral or gastric contents enter the apical segment of the right lower lobe. Anaerobes and gram negative organisms are the usual organisms in abscesses following aspiration. Cefuroxime + metronidazole
  • 13. This patient presented to Casualty with acute abdominal pain. What is the diagnosis? The slide shows pigmentation of the buccal mucosa in Addison's disease. Addison's disease (primary hypoadrenalism) commonly presents with non-specific systemic symptoms, including: abdominal pain; vomiting; weight loss; diarrhoea; malaise; fever and muscle cramps.
  • 14. What is the diagnosis? 1)Pseudoxanthoma elasticum 2)Acanthosis nigricans 3)Intertrigo 4)Neurofibromatosis 5)Addison's disease Acanthosis nigricans (pictured) has a characteristic hyperpigmented, velvety surface. It frequently occurs in the axillae, groins and in the skin fold of the neck and occasionally on the dorsum of the hand. Acanthosis nigricans is associated with: endocrine disease (acromegaly, Cushing's syndrome, insulin resistant diabetes mellitus), in polycystic ovary syndrome and as a paraneoplastic phenomenon (usually tumours of the GI tract, especially adenocarcinoma of the stomach).
  • 15. This 53-year-old woman presented to hospital with haematemesis. She had been hospitalised six months previously following a similar episode. What is the diagnosis? 1) Chronic hepatitis B 2) Ehlers-Danlos syndrome 3) Haemachromatosis 4) Pseudoxanthoma elasticum 5) Zollinger-Ellison syndrome The slide shows 'plucked chicken skin' appearance to skin folds in the neck. Elastic fibres in the skin degenerate and calcify, leading to this characteristic appearance. Patients are predisposed to peripheral and coronary arterial disease and to upper and lower gastrointestinal haemorrhage. Due to loss of elastic tissue, patients with pseudoxanthoma elasticum have an increased incidence of mitral regurgitation, aortic regurgitation and aortic dissection.
  • 16. A 46-year-old man presents with an extensive pruritic rash shown in picture A. 2 weeks previously he had a sore throat with the appearence shown in picture B. What is the cause of the rash? 1)Cutaneous T-cell lymphoma 2)Guttate psoriasis 3)Pituriasis Rosea 4)Pityriasis Rubra Pilaris 5)Scarlet fever Guttate psoriasis takes its name from the appearence of the lesions which are 'drop like'. It is a common presentation of psoriasis in children and young adults and may often follow and upper respiratory tract infection especially Streptococcal.Pityriasis Rosea is of unknown aetiology and is a widespread pink patchy rash that appears after a 'herald patch'.Pityriasis Rubra Pilaris is a papulosquamous disorder of unknown aetiology which presents as red-orange plaques. It is a cause of erythroderma.
  • 17. A 43 year-old HIV-positive man presented with this rash. He had been treated for Pneumocystis carinii pneumonia (PCP) six months previously when he presented for the first time. His current medication included highly active antiretrovial therapy (zidovudine (AZT), lamivudine (3TC) and nevirapine) and co-trimoxazole as PCP prophylaxis. His CD4 count was 210 cells/mm3 with an undetectable HIV viral load. What is the cause of his rash? 1)Co-trimoxazole allergy 2)Disseminated herpes simplex virus infection 3)Nevirapine hypersensitivity 4)Scabies 5)Varicella zoster virus The slide shows typical VZV (shingles). HIV-positive patients are more prone to VZV regardless of their CD4 count. In addition to the typical dermatomal distribution of the vesicular rash, HIV patients occasionally have vesicles scattered in adjecent dermatomes. In advanced HIV disease VZV can manifest as severe disseminated disease.
  • 18. This 29 year-old man presented to hospital with a four week history of progressively worsening dyspnoea on exertion. He also complained of a non-productive cough. Over the two days preceeding admission the patient had become breathless at rest and was started on oral co-amoxiclav by his general practitioner. On examination he was febrile 38°C and looked unwell. Candida was noted on the tonsilar pillars. Oxygen saturation was 95% on room air, but fell to 85% following a bout of coughing. No wheeze or crackles were heard in his chest. His chest radiograph is shown. What is the most likely diagnosis? 1)Pneumococcal pneumonia 2)Chlamydia pneumoniae pneumonia 3)Pneumocystis carinii pneumonia 4)Legionella pneumophilia pneumonia 5)Influenza A pneumonia The history is typical of PCP. Salient features are: several days/weeks of increasing dyspnoea; dry cough; marked oxygen desaturation with exercise. The presence of oropharyngeal candidiasis in a patient without a known immunosuppressive illness is highly suggestive of HIV/AIDS. The very brief course of antibiotics is not sufficient to explain the presence of Candida.
  • 19. What is the most likely visual field defect? 1)Arcuate 2)Central 3)Concentric 4)Lower quadrantic 5)Temporal The slide shows the typical appearance of toxocara retinitis with a lesion at the macula. In retinitis due to Toxocara canis, there is usually only a single, well demarcated lesion.
  • 20. This 58 year-old gentleman is 6' 6" tall and has small testicles. What is the likely karyotype? 1) 45 X 2) 46 XY 3) 47 XXX 4) 47 XXY 5) 47 XYY Picture shows gynaecomastia in a patient with a history suggesting Kilinefelter's syndrome. Klinefelter's is characterised by tall stature, small testes, azoospermia and gynaecomastia in a male. Plasma gonadotrophins are raised. Typical karyotype is 47XXY, though mosaics occur with 46XY/47XXY karyotype. There is an increased risk of breast cancer (20x higher than normal male).
  • 21. This 58 year-old gentleman is 6' 6" Which of the following investigation is most likely to reveal the diagnosis? 1)Testicular biopsy 2)Serum prolactin 3)Buccal smear 4)MRI scan of pituitary and hypothalamus 5)Plasma testosterone The slide shows gynaecomastia; the history (tall stature and small testes) suggests Klinefelter's syndrome. Buccal smear will be chromatin positive, indicating the presence of an extra X chromosome.
  • 22. This 52 year-old Greek woman presented with a right popliteal deep vein thrombosis. What is the most likely diagnosis? 1)Reitter's syndrome 2)Crohn's disease 3)Secondary syphilis 4)Pemphigus vulgaris 5)Behcet's disease Behcet's disease is a rare multisystem vasculitis commoner in males than females. Clinical features include the classical triad of (1) recurrent oral ulceration (2) recurrent genital ulceration (3) iritis. Other features include fever, abdominal pain, diarrhoea, episcleritis, polyarteritis, arterial and venous thromboses, and pericarditis. Patients may exhibit pathergy (developnment of pustules at venepuncture sites). HLA B5 is associated with ocular disease; HLA B12 is associated with recurrent oral ulcers. Treatment is with steroids or colchicine.
  • 23. A 48 year-old diabetic man presents with a 24 hour history of an increasingly painful left leg. What is the most likely causative organism? 1) Group A Streptococcus 2) Group B Streptococcus 3) Group G Streptococcus 4) Staphylococcus aureus 5) Staphylococcus epidermidis Ascending cellulitis is shown. Staphylococcus aureus & Streptococci are the commonest causative organisms. Group B Streptoccus has a prediliction for diabetic patients and is the likliest causative organism in this scenario.
  • 24. This patient with advanced HIV disease noted the appearance of these lesions on her face over several weeks What is the causative agent 1) Human papilloma virus 2)Human herpesvirus 8 3) Pox virus 4) Bartonella hensalae 5) Cryptococcus neoformans The slide shows the typical papules (with central umbilcation) associated with molluscum contagiosum. The condition is caused by infection with a pox virus and is seen in advanced HIV disease and AIDS. Bartonella hensalae is the bacterium that causes cat-scratch disease. Cryptococcus neoformans can cause cutaneous lesions in advanced HIV disease, but the appearance here is not typical of cryptococcosis.
  • 25. A 47 year-old lady presented two weeks after returning from a holiday in the Caribbean complaining of an intensely pruritic rash on her abdomen. What is the causative organsim? 1)Ancylostoma braziliense 2)Chironex fleckeri Southcott (Box Jelly Fish) 3)Borrelia burgdorferi 4)Onchocerca volvulus 5)Treponema pallidum The figure shows cutaneous larva migrans (creeping eruption). Usually caused by an animal hookworm, most commonly Ancylostoma braziliense. The infection is acquired by direct contact with dog or cat faeces – often acquired when sunbathing on contaminated sand, etc. The larvae burrow in the dermo-epidermal junction. Symptoms include pruritis and a raised, serpiginous erythematous rash that migrates at a rate of up to 1 cm/day. Treatment is with topical thiobendazole, or oral albendazole. Acute infection with the human nematodes Strongyloides stercoralis, Necator americanus and Ancylostoma duodenale, may produce a similar appearance.
  • 26. A 47 year-old man with advanced HIV disease presented with fever and obtundation. He has not attended his clinic appointments for several years and is not taking antiretroviral therapy. A CT scan of his brain was normal. A lumbar puncture was preformed; microscopy of his cerbrospinal fluid is shown below. What treatment should this patient receive? 1)Amphotericin B + flucytosine 2)High-dose intravenous cefotaxime 3)Sulfadiazine + pyrimethamine 4)High-dose intravenous aciclovir 5)Highly active antiretroviral therapy. The slide is an India ink stain of CSF that shows typical yeast-like forms of Cryptococcus sp.; the thick polysaccharide capsule is highlighted against the darker backgound of black ink. The history is characteristic of cryptococcal disease in AIDS. Treatment is with amphotericin B and flucytosine (5FC); patients then require lifetime suppression with fluconazole.
  • 27. The following 46-year-old man presents with exertional chest pain after walking about 100 metres on the flat. He is a non-smoker. An exercise tolerance test confirms the diagnosis of IHD. What is the most likely classification of his hyperlipidaemia? 1) Abetalipoproteinaemia 2) Chylomicronaemia syndrome 3) Familial hypercholesterolaemia 4) Familial hypertriglyceridaemia 5) Remnant hypercholesterolaemia The picture shows xanthelasma and corneal arcus. These are seen in common 'polygenic' hypercholesterolaemia, familial combined hyperlipidaemia and familial hypercholesterolaemia. Tendon xanthomata are seen in familial hypercholesterolaemia and remnant hypercholesterolaemia. Eruptive xanthomata are seen in chylomicronaemia syndrome and familial hypertriglyceridaemia. Striate xanthomata (plam creases) are seen in remnant hypercholesterolaemia.
  • 28. What is the diagnosis? 1) Erythema multiforme 2) Erythema marginatum 3) Erythema nodosum 4) Erythema gyratum repens 5) Erythema ab igne The slide shows typical target lesions of erythema multiforme.
  • 29. These are the shins of a 73 year old female who is admitted as a consequence of increasing confusion and inability to look after herself. Which one of the following investigations would be most likely to confirm the underlying diagnosis? 1) CT head scan 2) Plasma Glucose concentration 3) Thyroid function tests 4) Urea and electrolytes 5) Vitamin B12 The diagnosis is erythema ab igne and this is due to sitting too close to a fire. It frequently occurs on the front of the shins or lower back, the latter especially associated with the use of a hot water bottle. In this patient?s case the confusion and coldness with erythema ab igne suggest a diagnosis of hypothyroidism.
  • 30. This 52 year-old man was found collapsed on a golf course. Cardiopulmonary resuscitation was initiated. On arrival of the Paramedics, he was noted to be in ventricular fibrillation. What was the cause of his collapse? 1) Acute myocardial infarction 2) Acute rheumatic fever 3) Bacterial endocarditis 4) Cardiac Lyme disease 5) Lightening strike The figure shows a 'ferning' or 'arborescent' rash pathognomonic of a lightning strike. Also known as Lichtenberg figures. "The pathology of lightning, or keraunopathy, is known only to a few specialists."
  • 31. A 47-year-old Turkish man presented to casualty with this lesion on his arm. He also complained of discomfort in the left axilla. The lesion had developed over a period of two weeks and was painless. There was no past history of note. He had recently returned from a vacation to his parents’ goat farm in Turkey. What is the diagnosis? 1) Orf 2) Cutaneous anthrax 3) Pyoderma gangrenosum 4) Brucellosis 5) Plague The slide shows the typical appearance of cutaneous anthrax, caused by Bacillus anthracis. Cutaneous disease is the commonest form of the infection in humans and is usually due to contact with infected animals or animal products. Anthrax is endemic to herd animals in some parts of the world. Following exposure, the skin lesion evolves over a period of ~2 weeks into a papule, pustule, vesicle and eventually forms an ulcer with a central black eschar. The surrounding skin is usually boggy and oedematous. Lesions are usually painless with tender regional lymph nodes. Lesions heal spontaneously in 80- 90% of cases; 10-20% of patients progress and become bacteraemic – associated with a high mortality. Penicillin is effective in treating the infection.
  • 32. A 45 year-old dockyard worker presented with symptoms of fatigue and general malaise. He also reported developing ulcers on his hands after minor trauma. What is the most likely diagnosis? 1) Acute intermittent porphyria 2) Alcoholic haemosiderosis 3) Contact dermatitis 4) Haemachromatosis 5) Porphyria cutanea tarda Porphyria cutanea tarda (PCT) is associated with deficiency of hepatic uroporphyrinogen (URO) decarboxylase. The major clinical feature is cutaneous photosensitivity. Bullae develop on sun-exposed areas and lesions heal slowly, leaving scars. Porphyrins are increased in liver, plasma, urine and stool. Porphobilinigen (PBG) is normal. Factors contributing to PCT are alcohol (the commonest cause), excess iron and excess oestrogens.
  • 33. This 56 year old female presents with a 6 month history of weight loss and sweas. Examination reveals the appearances as shown, a pulse of 98 beats per minute and blood pressure is 120/70 mmHg. Investigations show a T4 of 25.3 nmol/l (NR 9.8 - 23), a TSH of 4.5 mU/l (NR 0.5 - 4.2 mU/l) and a T3 of 7.8 nmol/l (NR 3 - 6). Which of the following investigations would be most appropriate? 1) 123I Thyroid uptake scan 2) Beta-HCG concentration 3) CT ovaries 4) MRI headscan 5) Short synacthen test This patient has secondary hyperthyroidism with elevated TSH, T3 and T4. MRI headscan may be expected to demonstrate a pituitary macroadenoma. Alpha subunit is also secreted in large amounts and measurement of this should yield an elevated alphaSU: TSH ratio (usually 1:1). The diagnosis should be suspected when TSH concentrations are not suppressed in the presence of hyperthyroidism.
  • 34. This patient presented with a three month history of a gradually enlarging rash on her foot. She had a past history of rheumatoid arthritis. Urine dipstick was negative and a fasting blood glucose was normal. What is the most likely diagnosis? 1) Squamous cell carcinoma 2) Myxoedema 3) Neuropathic ulcer 4) Morphoea 5) Necrobiosis lipoidica diabeticorum Necrobiosis may prescede symptoms and signs of diabetes by several months.
  • 35. Which of the following statements is correct regarding this disease? 1) Immunofluorescent staining of a skin biopsy will show intercellular deposition of IgG at the dermal-epidermal junction 2) This disease is associated with internal malignancy 3) Faecal porphyrins will be elevated 4) Mucosal involvement is rare 5) This appearance is due to a cutaneous T-cell lymphoma The slide shows pemphigoid, a skin disease of the elderly (>60 years) characterised by the development of large blisters that heal without scarring. The condition is caused by IgG autoantibodies against components of the basement membrane. Histology shows IgG and C3 deposition along the basement membrane. Mucosal involvement is rare; if present it is generally associated with late disease and is seldom present at presentation.
  • 36. This 52 year old male is referred with reduced shaving frequency and libido of at least five years duration. Examination reveals the appearances as shown and he is noted to have a bitemporal hemianopia and slight galactorrhoea to expression. From the list select the most likely diagnosis. 1) Acromegaly 2) Klinefelter's syndrome 3) Macroprolactinoma 4) Microprolactinoma 5) Non-functional pituitary tumour This patient has typical features of hypopituitarism and the galactorrhoea suggests hyperprolactinaemia. In the presence of optic chiasmal compression the diagnosis is likely to be a macroprolactinoma. Although stalk compression with a non functioning tumour may cause hyperprolactinaemia the concentrations of prolactin are usually below 2000 mU/l and galactorrhoea would be rare. The patient does not appear acromegalic.
  • 37. This 82 year-old Jewish lady presented with this rash. What is the diagnosis? 1) Toxic epidermal necrolysis 2) Mycosis fungoides 3) Pemphigus vulgaris 4) Porphyria cutanea tarda 5) Pemphigoid Pemphigus vulgaris is a blistering disease seen in the elderly and is characterised by the formation of thin-walled blisters. Manual pressure applied to the skin may induce sloughing and separation of the epidermis (Nikolsky's sign).
  • 38. What does the slide show? 1) Intracapillary drusen 2) Grade 1 hypertensive retinopathy 3) Optic atrophy 4) Lipaemia retinalis 5) Papilloedema Papilloedema
  • 39. A 15-year-old boy was brought to casualty by his parents. He had been febrile and generally unwell for 2 days, but on the day of presentation had spiked a high fever, become slightly confused and started vomiting. A CT head scan was unremarkable and a lumbar puncture was performed. A Gram stain of his cerebrospinal fluid is shown. What is the diagnosis? 1) Haemophilus influenzae meningitis 2) Listeria meningitis 3) Meningiococcal meningitis 4) Pneumococcal meningitis 5) Viral meningitis The Gram stains shows Gram-positive diplococci, characteristic of Streptococcus pneumoniae.
  • 40. An 82-year-old man presents with weight loss (5kg) and a hoarse voice of two months’ duration. His chest radiograph is shown below. What clinical signs are likely to be found? 1) Dilated left pupil 2) Left ptosis 3) Inability to blink left eyelid 4) Inability to sweat on the right upper body 5) Right-sided exophthalmos The chest x-ray shows left upper lobar consolidation; the history of hoarseness implies involvement of the recurrent laryngeal nerve – most likely due to invasion by tumour. Upper lobar malignancies involving the superior pulmonary sulcus can destroy surrounding structures leading to a characteristic clinical pattern – Pancoast’s syndrome. The syndrome consists of pain in a C8-T2 distribution (caused by infiltration of these nerves) often accompanies by radiological evidence of destruction of the first and second ribs. Horner’s syndrome frequently co-exists due to infiltration of the sympathetic trunk. Horner’s syndrome consists of enophthalmos, ptosis, miosis and ipsilateral loss of the ability to sweat).
  • 41. A 27-year-old man with Down syndrome complains to his carers of intense perianal discomfort and pruritis. He lives in a long-term care facility and recently completed a seven-day course of antibiotics for a chest infection. Examination reveals perianal excoriation. A perianal swab is sent for microscopy (shown). What is the cause of his symptoms? 1) Clostridium difficile associated diarrhoea 2) Echinococcus granulosus 3) Enterobius vermicularis 4) Hymenolepis nana 5) Trichuriasis Enterobius vermicularis (threadworm) infection is not uncommon in children and in institution. Transmission is by the faeco-oral route and intense anal pruritis is the predominant symptom. Treatment is with mebendazole or piperazine. Hymenolepsis nana is a rodent cestode parasite that can be transmitted to humans and usually affects children. Abdominal pain, anorexia, diarrhoea, pruritis ani and urticaria are the most frequent symptoms. Eosinophilia may be present in heavy infestations. Treatment is with praziquantel. Echinococcus granulosus is the organism responsible for hydatid disease; it is a canine tapeworm. Trichuriasis is commoner in malnourished populations; symptoms are minimal but may result in growth retardation in children. Heavy burdens of infection may be associated with bloody diarrhoea; it is associated with rectal prolapse. Treatment is with mebendazole.
  • 42. A 9 year-old boy presents to casualty with visual disturbance. A visual field defect is detected on confrontational testing. Fundoscopy reveals the appearance shown below. What is the most likely cause of this appearance? 1) Cytomegalovirus 2) Medullated nerve fibres 3) Retinoblastoma 4) Toxocara canis 5) Toxoplasma gondii The slide shows the typical appearance of toxocara retinitis with a lesion at the macula. In retinitis due to Toxocara canis, there is usually only a single, well-demarcated lesion.
  • 43. What is the diagnosis in this 78 year-old man? 1) Pemphigus vulgaris 2) Dermatitis herpetiformis 3) Toxic epidermal necrolysis 4) Necrolytic migratory erythema 5) Pemphigoid Pemphigoid is a disease of the elderly (>60 years) characterised by the development of large blisters that heal without scarring. The condition is caused by IgG autoantibodies against components of the basement membrane. Blistering in pemphigoid occurs at the subepidermal level – deeper than the blisters of pemphigus vulgaris (which occur at the dermal-epidermal junction). Hence the tense blisters seen in pemphigoid; blisters are thin-walled and fragile in pemphigus – few intact blisters are ever seen.
  • 44. A 72 year old woman presents to A&E with fevers and pain in her right knee. On examination she is febrile 40°C; her right knee is swollen, hot and tender. She is unable to tolerate any passive movement of the joint. A knee aspirate is sent for microscopy – the Gram stain is shown below. What is the most appropriate management for this patient? 1) Treat for four weeks with intravenous benzyl penicillin + flucloxacillin 2) Treat for two weeks with intravenous vancomycin + rifampicin 3) Start antibiotics once sensitivities are known 4) Start prednisolone 40mg od 5) Surgical referral The slide shows Gram positive cocci growing in pairs and short chains – the diagnosis is Streptococcal septic arthritis. Septic arthritis has a high morbidity and mortality. Although antibiotics are clearly required, and should be given without delay, the patient needs an immediate orthopaedic surgical referral and should be taken for urgent joint washout. The duration of antibiotic therapy is judged by clinical response, but will usually be in the order of 4-6 weeks.
  • 45. This 19-year-old woman presented to the Accident & Emergency department complaining of painful legs. What is the most likely cause for these lesions? 1) Secondary syphilis 2) Cat scratch fever 3) Oral contraceptive pill 4) Sarcoidosis 5) Streptococcal infection Erythema nodosum is characterised by the presence of tender subcutaneous nodules, usually on the shins. Histology of these lesions shows a vasculitis of small venules and panniculitis. The commonest cause is Streptococcal infection. Other common common causes include: other upper respiratory tract infections; sarcoidosis, inflammatory bowel disease. Less common causes: TB, histoplasmosis, coccidioidomycosis, psittacosis, cat scratch fever, Yersinia infection, Salmonellosis, Chlamydial infection, drugs (oral contraceptive, sulfonamides, aspartame, bromides, iodides).
  • 46. This 58 year old lady presented with hypertension. What clinical sign is being demontrated in picture B? 1) Dehydration 2) Loss of subcutaneous fat 3) Skin fragility 4) Sensory peripheral neuropathy 5) Peripheral oedema Thin skin and loss of subcutaneous fat is a sign of Cushing's disease. It should raise the possibility of this as a diagnosis in individuals presenting with diabetes, hypertension, hypokalaemia, obesity or purpura. Cushing disease, the excess secretion of corticotropin from a pituitary adenoma that stimulates excess corticosteroid secretion from the adrenal glands, occurs primarily in women, aged 25 to 45 years. Easy bruisability is a commonly reported symptom. Orth and colleagues further describe the indications: "The skin is atrophic, the stratum corneum is thinned, and there is loss of subcutaneous fat. The skin becomes fragile and, in extreme cases, peels off with adhesive tape like damp tissue paper (Liddle's sign)."
  • 47. This 33-year-old man presented with a painful left eye. He was known to be HIV antibody positive. What is the causative agent? 1) Cytomegalovirus 2) Adenovirus 3) Human herpesvirus-8 4) Epstein-Barr virus 5) Pox virus The slide shows Kaposi's sarcoma (KS) of the eye. A typical KS lesion is visible on the left side of his forehead. KS can present with cutaneous or disseminated disease. Lesions on the head and neck, especially around the eyes, nose and mouth are common; they may be painful or painless.
  • 48. This 21 year old female presents with weight loss, weakness and dizziness. Which of the following antibodies would provide diagnostic information? 1) Anti- 21 Hydroxylase antibodies 2) Anti- Glutamic acid decarboxylase antibodies 3) Anti - Intrinsic factor antibodies 4) Anti- Thyroid peroxidase antibodies 5) Anti- Endomysial antibodies This patient has addison's disease as reflected by the pigmented appearance and the symptoms. Anti-21 hydroxylase antibodies are typically seen in a high percentage (80-90%) of such cases. Although other antibodies may be present, they are not as frequent.
  • 49. This is the chest X-ray of a 53 year old engineer presenting with increasing shortness of breath on exertion. What is the diagnosis? 1) asbestosis 2) byssinosis 3) siderosis 4) silicosis 5) talcosis The X-ray shows extensive pleural plaques from previous exposure to asbestos.
  • 50. EFFECT AGENT DISEASE Fibrosis, bullae, pneumothorax Alum.& al.oxide Aluminosis Pleural plaques, lung cancer,mesothelioma Asbestos Asbestosis Airway obstruction, loss of elasticity Cotton, flax, hemp Byssinosis Chemical pneumonitis Cadmium, Cobalt, Nickel, Zinc & others Metal fume fever Reversible airway obstruction Western Red Cedar and others Occupational Asthma Dust deposits Iron oxide Siderosis Dust deposits and fibrosis Silica Silicosis Perivascular fibrosis Talc, hydrated Mg. silicates Talcosis
  • 51. A 47 year-old man with advanced HIV disease presented with fever and obtundation. A CT scan of his brain was normal. A lumbar puncture was preformed; microscopy of his cerbrospinal fluid is shown . What is the diagnosis? 1) Primary central nervous system lymphoma 2) Pneumococcal meningitis 3) Toxoplasma encephalitis 4) Syphilitic meningitis 5) Cryptococcal meningitis Cryptococcus neoformans is a yeast-like fungus that can cause meningitis in patients with AIDS (CD4 < 50 cells/mm3) and other profoundly immunosuppressed patients. Symptoms are often mild initially with headache, slight neck stiffness and fever. However, patients can become obtunded rapidly. Diagnosis is by demonstration of C. neoformans in the CSF – shown on an India ink stain: the thick polysaccharide capsule is highlighted around the cell. CSF pressures can be very high – sometimes requiring daily therapeutic CSF tapping. Treatment is with amphotericin B and flucytosine (5FC); patients then require lifetime suppression with fluconazole.
  • 52. This 50 year old male presents with a three month history of tiredness, lethargy and weight gain. Six years previously he underwent trans-sphenoidal resection of a non-functional pituitary tumour. He currently takes hydrocortisone, thyroxine and testosterone therapy. Examination reveals the appearances as shown and a BMI of 30. Which of the following treatments would you consider for this patient? 1) Desmopressin 2) DHEAS 3) Fludrocortisone 4) Growth Hormone 5) Sibutramine This hypopituitary patient complains of tiredeness and weight gain despite adequate standard hormone replacement therapy. His current symptoms appear to be related to a GH deficiency and would benefit from GH therapy. He does not require DDAVP as there is no evidence of Diabetes Insipidus. Fludrocortisone is not required as adrenal function is preserved and mineralocorticoid activity which depends on the renin-angiotensin system and independent of pituitary function, is normal. Sibutramine which is licensed for the treatment of obesity would not be required.
  • 53. This pattern of inheritance is most consistent with ... 1) Haemophilia B 2) Hereditary spherocytosis 3) Sickle cell disease 4) Thalassaemia major 5) von Willebrand’s disease The pedigree chart shows a pattern of X-linked recessive inheritance. Sickle cell disease and thalassaemias are inherited in an autosomal recessive manner; hereditary spherocytosis and von Willebrand’s disease are inherited as autosomal dominant diseases. Examples of X-linked recessive diseases include: Haemophilia A & B, Duchenne muscular dystrophy, Becker muscular dystrophy, glucose-6-phosphate dehydrogenase deficiency, Lesch-Nyhan syndrome, nephrogenic diabetes insipidus, Hunter’s syndrome, Anderson-Fabry disease, ocular albinism.
  • 54. What is the diagnosis? 1) Anterior uveitis 2) Dislocation of the lens 3) Hyphaema 4) Hypopyon 5) Malignant melanoma of the iris The slide shows hyphaema, blood in the anterior chamber. Usually caused by trauma – often small objects (champagne corks, squash balls) hitting the eye. Aspiration may be required to prevent loss of vision.
  • 55. This patient was admitted to hospital with severe central abdominal pain. What name is given to this appearance? 1) Tophaceous gout 2) Eruptive xanthomata 3) Condyloma accuminata 4) Adenoma sebaceum 5) Chickenpox Typical lesions of eruptive xanthomata are shown: raised lesions, typically occurring on extensor surfaces. Eruptive xanthomata occur in hyperlipidaemic states associated with hypertriglyceridaemia. Well- recognised associations of hypertriglyceridaemia include acute pancreatitis and lipaemia retinalis.
  • 56. What is the diagnosis? 1) Anterior uveitis 2) Dendritic ulcer 3) Rubeosis iridis 4) Episcleritis 5) Scleromalacia perforans Dendritic ulcers are caused by Herpes simplex virus. Presentation is usually with pain, photophobia, blurred vision , conjunctivitis and chemosis. Steroid eye drops are contraindicated as they may induce massive amoeiboid ulceration and blindness. Diagnosis is by instillation of fluorescein eye drops which stain the ulcer (shown in slide).
  • 57. What is the most likely diagnosis in this 22 year old woman with a history of chronic sinusitis? 1) Brain abscess 2) Cerebral toxoplasmosis 3) Glioblastoma multiforme 4) Metastatic carcinoma 5) Histoplasmosis Cerebral metastases, toxoplasmosis, histoplasmosis and some primary brain tumours can appear as ring-enhancing lesions on a CT brain. On balance, the most likely diagnosis with a slitary frontal lesion in a young patient with chronic sinusitis is a frontal brain abscess.
  • 58. A 35 year-old man with HIV disease presents with sudden loss of vision in his right eye. He is not curently taking antiretroviral therapy. A recent CD4 count was measured at 20 cells/mm3. What is the most appropriate therapy to initiate? 1) High-dose intravenous aciclovir 2) Intravenous ganciclovir 3) Zidovudine + lamivudine + nevirapine 4) Sulfadiazine + pyrimethamine 5) Amphotericin B + flucytosine The slide shows the typical 'cottage cheese and tomato ketchup' or 'pizza' appearance of CMV retinitis in a patient with HIV disease. Intravenous ganciclovir is currently the treatment of choice. Available forms of ganciclovir include intravenous and oral preparations as well as an ocular implant. The principal disadvantages of the latter two are: the poor bioavailability of the oral preparation; the implant is effective at clearing ocular disease, but CMV is disseminated at this stage of AIDS and the implant does not address systemic infection. Pyrimethamine + sulfadiazine are used to treat toxoplams retinitis; zidovudine + lamivudine + nelfinavir constitutes highly active antiretrovral ('combination') therapy (HAART) for HIV disease. Initiation of HAART has been shown to worsen active CMV eye disease and should be witheld until CMV is in remission.
  • 59. This 80 year-old lady presented with congestive cardiac failure. What is the diagnosis? 1) Scurvy 2) Ricketts 3) Paget's disease of bone 4) Syphilis 5) Achondroplasia Typical appearance of Pagetic tibiae is shown, with characteristic bowing. Complications of Paget's disease include: bone pain, pathological fractures, nerve deafness, spinal cord compression, high-output cardiac failure and osteosarcoma. The latter affects ~10% of patients who have had the disease for over 10 years. Bowing of the tibia may also be a feature of ricketts, syphilis and achondroplasia.
  • 60. This 34 year-old gay man was referred to the hospital by his dentist with a painful oral lesion. What is the infectious agent that caused this appearance? 1) Candida albicans 2) Human herpesvirus-8 3) Epstein-Barr virus 4) Herpes simplex virus 5) Treponema pallidum The picture shows characteristic purple lesions of Kaposi's sarcoma (KS) on the hard palate; it is seen almost exclusively in gay men with HIV disease. In HIV infection, KS lesions occur most commonly on the face; however lesions may be widely disseminated in skin, bronchial tree and G-I tract. KS can occur at any time during the course of HIV disease, irrespective of CD4 count; disseminated disease is commoner in late stage HIV disease/AIDS. In all cases, KS is caused by human herpesvirus 8 infection (HHV-8), probably sexually acquired.
  • 61. This patient has an abnormal chest x-ray. What is the most likely diagnosis? 1) Amyloidosis 2) Sarcoidosis 3) Secondary syphilis 4) Squamous cell carcinoma 5) Tuberculosis The slide shows cutaneous tuberculosis (lupus vulgaris). Cutaneous TB usually occurs due to spread from an endogenous source; >80% of cases occur on the face and neck. Lesions begin as papules and coalesce to form a plaque. The centre of the lesion consists of scar tissue while the lesion extends from the periphery; apple-jelly nodules are classically described at the margins of the lesions.
  • 62. What is the diagnosis? 1) Rheumatoid arthritis 2) Osteoarthritis 3) Tophaceous gout 4) Sarcoidosis 5) Psoriatic arthropathy The slide shows the typical appearance of chronic tophaceous gout.
  • 63. Immunofluorescent staining of a skin biopsy from this patient showed intercellular deposition of IgG at the dermal-epidermal junction. What is the diagnosis? 1) Staphylococcal scalded skin syndrome 2) Necrolytic migratory erythema 3) Pemphigoid 4) Dermatitis herpetiformis 5) Pemphigus vulgaris Pemphigus vulgaris is a blistering disease seen predominantly in elderly patients. It is characterised by the formation of thin- walled blisters that rupture easily. Immunofluorescent staining of a biopsy sample shows deposition of IgG on acanthocytes at the dermal-epidermal junction – resulting in a 'crazy paving' effect.
  • 64. A 19-year-old man presented with pleuritic chest pain which occurred suddenly while playing football. He presented to the Accident & Emergency department complaining of dyspnoea. What is the definitive treatment for this condition? 1) High-flow inspired oxygen 2) Low molecular weight heparin 3) Nebulized salbutamol 4) Intravenous amoxycillin + clarithromycin 5) Intercostal chest drain insertion The slide shows a large left sided pneumothorax with midline shift (tension pneumothorax). An intercostal chest drain should be inserted.
  • 65. Histologically, these lesions are characterized by deposition of IgG and C3 along the basement membrane. What is the diagnosis? 1) Pemphigoid 2) Staphylococcal scalded skin syndrome 3) Mycosis fungoides 4) Pemphigus vulgaris 5) Porphyria cutanea tarda The slide shows pemphigoid, a skin disease of the elderly (>60 years) characterised by the development of large blisters that heal without scarring. The condition is caused by IgG autoantibodies against components of the basement membrane. Histology shows IgG and C3 deposition along the basement membrane.
  • 66. A 32 year-old man presented to hospital with a four week history of progressively worsening dyspnoea on exertion. He also complained of a non-productive cough. Over the two days preceeding admission the patient had become breathless at rest and was started on oral co- amoxiclav by his general practitioner. On examination he was febrile 38°C and looked unwell. Candida was noted on the tonsilar pillars. No wheeze or crackles were heard in his chest. His chest radiograph is shown. Which of the following is most likely to assist in confirming the diagnosis? 1) Peak expiratory flow rate 2) Oxygen saturations pre- and post-excercise 3) Legionella urinary antigen 4) Sputum culture 5) Blood pressure measured in inspiration and expiration The history is characteristic of Pneumocystis carinii pneumonia (PCP). The presence of oropharyngeal candidiasis in a patient without a known immunosuppressive illness is highly suggestive of HIV/AIDS. Salient features are: several days/weeks of increasing dyspnoea; dry cough; marked oxygen desaturation with exercise. This can be done on an exercise bike, or by asking the patient do do some simple exercise on the ward (sit back and forward in bed several times; squat and stand several times; walk a flight of stairs, etc.)
  • 67. What is the diagnosis? 1) Erythema annulare 2) Erythema nodosum 3) Erythema multiforme 4) Erythema marginatum 5) Erythema migrans The slide shows typical target lesions of erythema multiforme. "EM, first described in 1860 by von Hebra, is characterized by symmetrical target- like lesions. These lesions are described as erythematous papules surrounded by a raised, erythematous ring, which is in turn encircled by an erythematous outer ring. The lesions are generally located on the extremities"
  • 68. What is the diagnosis? 1) Medullated nerve fibres 2) Central retinal vein thrombosis 3) Pre-proliferative diabetic retinopathy 4) Papilloedema 5) Angioid streaks The slide shows papilloedema. "Management of optic disc edema begins with a correct diagnosis. Most importantly, it is crucial to distinguish between papilledema and the many other forms of optic disc edema, including 'masqueraders' such as buried optic disc drusen. Consider the acuity, visual fields, ophthalmoscopy findings and especially the laterality of presentation carefully in the initial work-up."
  • 69. A 45-year-old woman presented to the Accident & Emergency department with a high spiking fever ten days after returning from holiday in Kenya. Before leaving the U.K. her General Practitioner had prescribed mefloquine malaria prophylaxis. She had started the mefloquine one week before travelling and had continued to take it while she was in Africa. She complained of regular spiking fevers and rigors and of lethargy and malaise. Physical examination was unremarkable apart from a temperature of 40°C. A peripheral blood film is shown below. What is the diagnosis? 1) African trypanosomiasis 2) Dengue fever 3) oa loa 4) Onchocerciasis 5) Plasmodium falciparum malaria The incubation peroid for falciparum malaria is approximately 12 days. Chemoprophylaxis should be started 1 week before travelling to a malaria-endemic country and continued for 1 month after returning. Severe malaria is defined parasitologically as >5% of rbcs parasitised. However, the degree of parasitaemia on the peripheral film is usually a gross underestimate of the true parasite burden, as only younger ring forms are seen in the peripheral circulation in falciparum malaria. In the slide shown, the blood film shows ring forms within erythrocytes; some erythrocytes contain 2-3 parasites per cell – typical of falciparum; other forms of malaria seldom have >1 parasite per red cell.
  • 70. The pedigree chart below is that of a family in which there is a history of young adults presenting with subacute bilateral visual loss. What are the chances of the individual labelled ‘A’ being affected by the disease? 1) 0% 2) 25% 3) 50% 4) 75% 5) 100% The clinical description is that of Leber’s hereditary opic neuropathy, one of the first diseases to be aetiologically linked with inheritance of mitochndrial DNA. Mitochodrial DNA is always inherited from the mother and diseases therefore show a distinctive maternal pattern of transmission.100%
  • 71. This is the coronary angiogram of a 34 year old male who presents with chest pain of 2 hours duration. ECG demonstrates widespread ST segment depression throughout the chest leads. Investigations reveal a cholesterol concentration of 4.5 mmol/l, a triglyceride concentration of 2.0 mmol/l, a fasting glucose of 4.2 mmol/l and an ESR of 10 mm/hr with a CRP of 4.5 mg/l. Troponin T concentration is elevated. From the list, what is the most likely diagnosis. 1) Aortic dissection 2) Giant Cell Arteritis 3) Kawasaki Disease 4) Polyarteritis Nodosa 5) Takayasu's disease This patient presents with a premature acute coronary syndrome. However, his lipid profile is entirely normal as is glucose and inflammatory parameters are normal arguing against any inflammatory process. The coronary angiogram reveals out-pouching (aneurysms) and would be typical of previous Kawasaki disease. This is a childhood febrile illness which results in inflammation of the mucous membranes with a desquamative skin rash. The condition is frequently self-limiting but it is important to recognise as it causes coronary arterial inflammation resulting in aneurysm formation (25% of cases) which may present much later in life. Coronary disease can be prevented with treatment which includes NSAIDs and gammaglobulin infusion.
  • 72. The structure shown below was identified on microscopy of a sputum sample from a patient who presented with haemoptysis. He has an abnormal chest radiograph. What treatment should be started? 1) Intravenous amphotericin B 2) Intravenous amoxycillin/clavulanic acid + clarithromycin 3) Intravenous cefotaxime 4) Intravenous vancomycin 5) Isoniazid + rifampicin + ethambutol + pyrazinamide The slide shows the typical morphology of Aspergillus fumigatus. The patient will need treatment with amphotericin.
  • 73. A 9 year-old boy presents to casualty with visual disturbance. A visual field defect is detected on confrontational testing. Fundoscopy reveals the appearance shown below. What is the most likely cause of this appearance? 1) Cytomegaloviru 2) Medullated nerve fibres 3) Retinoblastoma 4) Toxocara canis 5) Toxoplasma gondii The slide shows the typical appearance of toxocara retinitis with a lesion at the macula. In retinitis due to Toxocara canis, there is usually only a single, well- demarcated lesion.
  • 74. This 25 year old man presented with a four day history of fever, malaise and a pruritic rash. What is the diagnosis? 1) Meningiococcal septicaemia 2) Disseminated gonococcal infection 3) Chickenpox 4) Guttate psoriasis 5) Scabies A pruritic vesicular rash. Note traces of calamine lotion. Chickenpox
  • 75. A 77-year-old man presents with a headache. Both pupils react equally to light and accommodation. The abnormality shown is probably caused by 1) A complete lesion of the left oculomotor nerve nucleus in the midbrain 2) Compression of the right third nerve by a posterior communicating artery aneurysm 3) Compression of the left third nerve by an anterior communicating artery aneurysm 4) Compression of the fourth cranial nerve in the cavernous sinus 5) Microinfarction of the left third nerve The third (oculomotor) nerve nucleus complex lies in the midbrain. Motor neurones project to the ipsilateral medial rectus, inferior rectus, and internal oblique muscles and to the contralateral superior rectus. One central nucleus innervates levator palpebrae superioris bilaterally. Therefore a midbrain infarct that destroys the central nucleus will result in bilateral ptosis. Damage to the oculomotor nerve during its course results in ipsilateral ptosis; at rest, the globe is diverted downwards and laterally. The eye cannot be moved medially, superiorly or inferiorly. The effect on the pupil is variable, depending on the location of the lesion. Compression of the nerve (e.g. by tumour, posterior communicating or posterior cerebral artery aneurysms) results in an acute total (painful) third nerve palsy with a dilated unreactive pupil. Pupillary dilatation occurs early when the nerve is compressed since sympathetic nerve fibres that innervate the iris are carried on the outside of the nerve bundle. Pupillary sparing is characteristic of third nerve lesions caused by infarction in patients >50 years of age with diabetes or hypertension.
  • 76. This 52 year old female presents with a one day history of fever and this facial appearance. The patient is penicillin allergic having had a problems with facial swelling previously. Which of the following antibiotics would be the most appropriate treatment for this patient? 1) Cefuroxime 2) Chloramphenicol 3) Co-trimoxazole 4) Erythromycin 5) Vancomycin Erysipelas is a Streptococcus pyogenes infection of the deep dermis and subcutis. Treatment relies upon IV antibiotics such as Benzylpenicillin and Erythromycin. In a penicillin allergic patient clearly a macrolide is the drug of choice from the above list. There is a 10% cross allergy between cephalosporins and penicillins. Although the organism is likely to be sensitive to both vancomycin and chloramphenicol, the latter is used with limitations due to the incidence of aplastic anaemia and the former tends to have restricted use due to nephrotoxicity. Complications include sepsis, cerebral abscess and venous sinus thrombosis.
  • 77. What is the diagnosis? 1) Dermatomyositis 2) Systemic sclerosis 3) Peutz-Jegher's syndrome 4) Mixed mitral valve disease 5) Facial lipodystrophy Systemic sclerosis is a multisystem disease of unknown aetiology that is characterised by multi-organ fibrosis. The disease affects blood vessels, skin, G-I tract, heart, kidneys and lungs. The CREST syndrome (calcinosis, Raynaud's phenomenon, esophageal dysmotility, sclerodactyly, telangiectasia) is a localised form of the disease generally associated with a better prognosis. The distinction between CREST and SSc cannot be made on the basis of a picture alone.
  • 78. The plain x-ray of this man's hands does not show any evidence of bone erosion. Which of the following diseases is most likely to cause this appearance? 1) Parkinson's disease 2) Rheumatoid arthritis 3) Gouty arthropathy 4) Osteoarthritis 5) Reiter's syndrome The diagnosis is Jaccoud's arthropathy. The picture shows joint subluxations and swan neck deformities, caused by recurrent episodes of synovitis that damage tendon sheaths and slings resulting in joint deformity. Jaccoud's arthropathy is seen in: SLE; rheumatic fever, Parkinson's disease; hypocomplementaemic urticarial vasculitis.
  • 79. A 73-year-old lady presents with anaemia and the following blood picture. Hb 8.5 g/dL MCV 69 fL WCC 5.2 x109/L Plt 240 x109/L Physical examination reveals the abnormality shown in the picture. OGD and colonoscopy show no other type of lesion. She was transfused with 4 units of packed red cells. What is the best course of subsequent management? 1) Argon beam ablation therapy 2) Genetic counselling 3) Nasal skin graft 4) Mefanamic acid 5) Oral iron therapy The slide shows the typical appearance of Hereditary Haemorrhagic Telangiectasia - an autosomal dominant condition associated with bleeding (usually nose and GI tract) from early teens and worsening after the age of 50. There may also be AV malformations of the brain, lung and GI tract. Argon ablation is used for Gastric antral vascular ectasia (GAVE) (Gastroenterology 1984;87:1165-70). Nasal skin grafts are used for persistent epistaxis but this lady has telangiectasia in many other places. Oestrogen therapy can be effective especially in women but the evidence is not strong. Lancet (1990;335:953-5) Oral iron therapy
  • 80. This patient presented following a collapse at home. He was noted to be in first degree heart block on admission to Casualty. What is the diagnosis? 1) Acromegaly 2) Hypopituitarism 3) Myasthenia gravis 4) Myotonic dystrophy 5) Parkinson's disease Myotonic dystrophy is the commonest adult muscular dysrophy. Features include frontal baldness in men, atrophy of temporalis, masseters and facial muscle. Neck muscles (including sternocleidomastoid) are involved early in the course of disease. Cardiac abnormalities are common, and include first degree heart block and complete heart block.
  • 81. This blood film was obtained on a 37 year old male. What is the likely cause for these abnormalities? 1) Disseminated Intravascular Coagulation 2) Hereditary spherocytosis 3) Malaria 4) Splenectomy 5) Trypanosmiasis The image shows Howell-Jolly bodies. These are spherical bluish inclusions within erythrocytes and are nuclear fragments of condensed DNA which are normally removed by the spleen. They are seen in severe haemolytic anaemias or in dysfunctional/asplenic patients.
  • 82. A 16 year-old boy is admitted to hospital for investigation of a chronic cough. He has a history of recurrent episodes of bronchitis since childhood and missed a significant amount of school. He also gives a longstanding history of loose stools – which he describes as being 'greasy'. He lives with his parents on a dairy farm. He has two brothers who are both well. On examination he appears small for his age and weighs 50kg. Loss of the nail angle is noted in all digits. There are widespread coarse crackles audible in both lung fields, most notable at the bases. A sputum sample is sent for culture: the culture plate is shown. What is the most likley diagnosis? 1) Chronic granulomatous disease 2) Cystic fibrosis 3) Hypogammaglobulinaemia 4) Tuberculosis 5) Extrinsic allergic alveolitis The history is most suggestive of CF (chronic cough, steatorrhoea due to pancreatic exocrine insufficiency). The culture plate shows a growth of Pseudomonas aeruginosa, characterized by the green colouration of the colonies - due to production of the pigment pyocyanin.
  • 83. Which of the following statements regarding this disease is correct? 1) Immonufluorescent staining of a biopsy specimen will show IgG and C3 deposition along the basement membrane 2) This appearance is due to a cutaneous T-cell lymphoma 3) Faecal porphyrins will be elevated 4) Mucosal involvement is rare 5) Systemic corticosteroids are the mainstay of treatment The slide shows the characteristic rash of pemphigus vulgaris. Blisters are thin-walled and rupture easily (intact blisters are rarely seen). Large surface area of the body can be affected and the mortality without treatment is high. Mucosal involvement is common at presentation (unlike pemphigoid). Treatment is with high-dose corticosteroids.
  • 84. This thoracic CT scan was performed on Brazilian a man who presented with dysphagia. What is the most likely causative organism? 1) Treponema pallidum 2) Trypanosoma cruzi 3) Mycobacterium tuberculosis 4) Leishmania donovani 5) Tropheryma whipelii The slide shows a chagasic megaoesophagus: a massively dilated oesophagus containing both food residue and radio-opaque contrast medium. Chagas disease is cause by T. cruzi, which infects humans via a bloodsucking insect vector (triatomine bugs).
  • 85. A 21-year-old woman presented with a four-week history of increasing exertional dyspnoea and a dry cough. Her chest was clear to auscultation and she was afebrile. Her chest radiograph is shown below. Investigations: Hb 13.2g/dl WBC 3.9x109/l Neutrophils 2.5x109/l Lymphocytes 1.0x109/l Monocytes 0.4x109/l Platelets 390x109/l Sodium 141mmol/l Potassium 4.7mmol/l Urea 5.5mmol/l Creatinine 102mmol/l Calcium 2.6mmol/l Bilirubin 8µmol/l Alkaline phosphatase 110µmol/l AST 22iu/l Total protein 55g/dl Globulin 31g/dl What is the most likely diagnosis? 1) Chlamydia pneumoniae pneumonia 2) Lymphoma 3) Pneumocystis carinii pneumonia 4) Sarcoidosis 5) Wegener’s granulomatosis click
  • 86. The chest x-ray shows bilateral reticulonodular shadowing and bilateral hilar lymphadenopathy. There is lymphopoenia, hyperglobulinaemia and hypercalcaemia. Calculation of corrected calcium: Add 0.1 mmol/l of calcium for every 4g/dl that the albumin level is below 40 g/dl Albumin + globulin = Total protein Albumin = Total protein – globulin = 55 - 31 = 24 Corrected calcium = 2.6 + (((40-24)/4) x 0.1) = 2.6 + ((16/4) x 0.1) = 2.6 + (4 x 0.1) = 2.6 + 0.4 = 3.0 mmol/l
  • 87. What is the diagnosis? 1) Guttate psoriasis 2) Scabies 3) Stevens-Johnson syndrome 4) Chickenpox 5) Eosinophilic folliculitis The slide shows chickenpox in an adult. Chickenpox is caused by varicella zoster virus. Infection most commonly affects children aged 5-9 years. Patienst are infectious for ~48 hours prior to the appearance of the rash and until all lesions have crusted. Presents as a low-grade fever and malaise for 3-5 days. Skin lesions (macules, papules and vesicles) vary in stages of evolution with successive crops over 2-4 days.
  • 88. Which of the following is not a recognized complication of this condition? 1) Marked joint laxity 2) Acute pancreatitis 3) Angioid streaks of the fundus 4) Accelerated atherosclerosis 5) Gastrointestinal haemorrhage The slide shows the typical 'plucked chicken skin' appearance of pseudoxanthoma elasticum.Marked joint laxity is there
  • 89. This gentleman first presented in his early 20's with haemoptysis. He has required intermittent blood transfusions throughout his life. What is the diagnosis? 1) Hereditary haemorrhagic telangiectasia 2) Wegener's granulomatosis 3) Kaposi's sarcoma 4) Hepatitis C 5) Peutz-Jegher's syndrome Typical telangiectasia of hereditary haemorrhagic telangiectasia (also known as Osler-Weber-Rendu syndrome) are seen. The condition is inherited as an autosomal dominant trait; sporadic cases also occur. Patients typically present early in adult life with anaemia due to occult bleeding from gastrointestinal telangiectasiae. Lesions commonly occur in the mouth and on the face. Complications of the disease include haemoptysis and pulmonary haemorrhage.
  • 90. This 42 year-old lady presented with a grand mal seizure. CT scan of her head showed several small areas of periventricular calcification. What is the diagnosis? 1) Neurofibromatosis 2) Tuberose sclerosis 3) von Hippel-Lindau syndrome 4) Multiple endocrine neoplasia type IIb 5) Gardner's syndrome The picture shows subungual fibromas, characteristic of tuberose sclerosis. Fibromas may also develop within the CNS, where they calcify. Other manifestations of tuberose sclerosis include Ash-leaf macules (discrete areas of hypopigmentation; often present at birth), shagreen patch (rough patch of skin with a texture similar to shark skin), adenoma sebaceum (angiofibromas of the face).
  • 91. This HIV positive man presented with cough, haemoptysis and a rash (shown). What is the causative agent? 1) Mycobacterium tuberculosis 2) Pneumocystis carinii 3) Cytomegalovirus 4) Epstein-Barr virus 5) Human herpesvirus-8 The picture shows characteristic purple lesions of Kaposi's sarcoma (KS); it is seen almost exclusively in gay men with HIV disease. In HIV infection, KS lesions occur most commonly on the face; however lesions may be widely disseminated in skin, bronchial tree and G-I tract. Pulmonary disease can present with dyspnoea and cough with haemoptyis; it is usually associated with cutaneous disease, but can present in isolation. Chest x-ray may show pulmonary nodules. KS is caused by human herpesvirus-8 (HHV-8) infection.
  • 92. This 52 year old female presents with a two day history of fever and painful facial rash. What is the likely diagnosis? 1) Erysipelas 2) Impetigo 3) Lupus pernio 4) Lupus Vulgaris 5) Rosacea Erysipelas is a streptococcal pyogenes infection of the deep dermis and subcutis. Treatment relies upon IV antibiotics such as Benzylpenicillin and Erythromycin. Complications include sepsis, cerebral abscess and venous sinus thrombosis.
  • 93. Which of the following statements are correct regarding this patient's condition? 1) It occurs more commonly in men 2) Rheumatoid factor is positive in >90% of cases 3) It is associated with an erosive arthritis 4) Raynaud's phenomenon is a feature in ~10% 5) It is associated with a reduced transfer factor The slide shows a patient with systemic sclerosis (SSc). Features present include microstomia, facial telangiectasia and beaking of the nose. Rheumatoid factor is positive in 25% of cases; positive ANF in 95%. Autoantibodies strongly associated with SSc include antinucleolar antibodies and anti-U3 nucleolar RNP. Anticentromere antibodies are strongly associated with CREST syndrome. The CREST syndrome (calcinosis, Raynaud's phenomenon, esophageal dysmotility, sclerodactyly, telangiectasia) is a localised form of SSc generally associated with a better prognosis. The distinction between CREST and SSc cannot be made on the basis of a picture alone. It is associated with a reduced transfer factor
  • 94. You are called to see a 58-year-old lady on the ward because the nursing staff have noticed the following abnormality of her eye. She is an inpatient with a history of paroxysmal atrial fibrillation and rheumatoid arthritis. The lady complains only of a minor ache in the eye. Which of her medication is a likely cause of the lesion? 1) Amiodarone 2) Aspirin 3) Folic acid 4) Meloxicam 5) Methotrexate The picture shows a subconjunctival haemorrhage which is an alarming adverse effect of aspirin therapy (and other antiplatelets). It usually resolves over 10 - 14 days. If the haematoma is large it may be worth considering prophylactic antibiotic eyedrops.
  • 95. The genetic tree shown is that of a family in which there is a history of cystic fibrosis. Assuming that the gene frequency in the general population is 1/20, what is the chance of individual ‘A’ having the disease? 1) 1/20 2) 1/80 3) 1/120 4) 1/160 5) 1/320 Cystic fibrosis is inherited as an autosomal recessive disease. In this case, since both parents are unaffected they must both be carriers in order to have a child with CF. The chances of the mother being a carrier is 2/3 (the mother's parents must both be carriers since she has a brother and a sister with the disease, and since she is unaffected she must either be a carrier or 'normal'). We must assume that the chance of the child's father being a carrier is 1/20 (the gene frequency in the general population); and the chances of two carriers of the CF gene having a child with the disease is 1/4. Therefore, the chance of individual 'A' being affected is: 2/3 x 1/4 x 1/20 = 1/120
  • 96. This 72 year-old lady presented with dysphagia. Which of the following autoantibodies is highly specific for this condition? 1) Rheumatoid factor 2) Anticardiolipin antibody 3) Antinuclear factor 4) Anticentromere antibody 5) Antimitochondrial antibody The slide shows the hands of a patient with systemic sclerosis/CREST (the distinction cannot be made on the basis of the picture alone). Rheumatoid factor is non-specific and is positive in 25% of cases; ANF is positive 95%. Autoantibodies strongly associated with SSc include antinucleolar antibodies and anti-U3 nucleolar RNP. Anticentromere antibodies are strongly associated with CREST syndrome. Anti-SSA (Ro) and anti-SSB (La) antibodies are seen in SSc associated with Sjogren’s syndrome. Anticardiolipin antibodies may be found in association with SLE. Antimitochondrial antibodies are associated with primary biliary cholangitis. Anti-RNP antibodies are also associated with mixed connective tissue disease.
  • 97. This patient was brought in to hospital unresponsive. Which of the following is the most appropriate initial step in management? 1) CT head scan 2) Blood cultures 3) Full blood count 4) Lumbar puncture 5) Administration of intravenous cefotaxime The picture shows a purpuric rash of meningiococcal septicaemia. All options are relevant in the overall management of a patient with meningiococcal disease. However, administration of iv antibiotics is the greatest priority – regardless of whether cultures have been sent.
  • 98. An 82-year-old man presents with weight loss (5kg) and a hoarse voice of two months’ duration. His chest radiograph is shown below. What clinical signs are likely to be found? 1) Dilated left pupil 2) Left ptosis 3) Inability to blink left eyelid 4) Inability to sweat on the right upper body 5) Right-sided exophthalmos The chest x-ray shows left upper lobar consolidation; the history of hoarseness implies involvement of the recurrent laryngeal nerve – most likely due to invasion by tumour. Upper lobar malignancies involving the superior pulmonary sulcus can destroy surrounding structures leading to a characteristic clinical pattern – Pancoast's syndrome. The syndrome consists of pain in a C8-T2 distribution (caused by infiltration of these nerves) often accompanies by radiological evidence of destruction of the first and second ribs. Horner's syndrome frequently co-exists due to infiltration of the sympathetic trunk. Horner's syndrome consists of enophthalmos, ptosis, miosis and ipsilateral loss of the ability to sweat).
  • 99. This 34 year old female presents with a nine month history of weight gain, weakness and amenorrhoea. Examination reveals the appearances as shown, a blood pressure of 180/110 mmHg and proximal myopathy. Urinalysis shows ++ glucose and a pregnancy test is negative. Which of the following would be the most appropriate diagnostic test? 1) Random Cortisol concentration 2) Oestradiol concentration 3) Oral glucose tolerance test 4) Pituitary MRI scan 5) Urine free cortisol estimation This patient appears typically Cushingoid. She is also hypertensive, has glycosuria suggesting diabetes and has a proximal myopathy. The diagnosis is likely to be Cushing's syndrome and the most appropriate investigation would be urine free cortisol estimation. Elevated UFC has high sensitivity and specificity (above 95%) for the diagnosis of Cushing's syndrome. Another useful screening test would be the 1mg overnight dexamethasone suppression test which has similar sensitivity and specificity. Features that would argue against pseudo-cushing's due to obesity or PCOS would be proximal myopathy, easy bruising and thin skin. Amenorrhoea is pften associated and is due to hypogonadotrophic hypogonadism.
  • 100. What is the diagnsosis? 1) Tuberculoid leprosy 2) Pyoderma gangrenosum 3) Morphoea 4) Necrobiosis lipoidica dibeticorum 5) Pre-tibial myxoedema Necrobiosis lipoidica is associated with diabetes mellitus. It is commoner in females than males. The lesions are painless.
  • 101. This patient presents with a sudden onset of frontal headache, diplopia and vomiting. Examination reveals the appearances as shown. There is also a constriction of outer visual fields in both eyes, a loss of sensation over the left/right upper maxilla. Investigations reveals a sodium of 128 mmol/l, a potassium 4.2 mmol/l, a urea of 4.2 mmol/l and a creatinine of 67 micromol/l with a plasma glucose of 3.8 mmol/l. Which of the following diagnoses is most likely: 1) Anterior communicating artery aneurysm rupture 2) Lateral Medullary syndrome 3) Meningitis 4) Pituitary apoplexy 5) Weber's syndrome This patient has a sudden onset headache and features of a cavernous sinus compression but with compression of the chiasm as suggested by the bitemporal hemianopia. This suggests pituitary apoplexy. The mild hyponatraemia may be a consequence of the either SIADH or secondary hypoadrenalism. Pituitary apoplexy usually occurs in a pre-existent pituitary tumour which may be entirely asymptomatic before presentation. Weber’s syndrome is a mid brain stroke with ipsilateral third nerve palsy and contralateral hemiplegia. Wallenberg's syndrome or lateral medullary syndrome is due to infarction of the posterior inferior cerebellar artery resulting in ipsilateral V, IX, X nerve involvement, dissociated sensory loss and ataxia.
  • 102. A 68 year-old man presented to casualty complaining of high fevers and night sweats two weeks after discharge from hospital following a left knee replacement. Of note, he had undergone splenectomy 20 years previously after he suffered splenic rupture in a road traffic accident. On examination, his left knee appeared swollen and was hot to touch. Aspirate of the joint was sent for microscopy. Gram's stain of the aspirate is shown below. What is the causative organism? 1) Candida albicans 2) Enterococcus faecalis 3) Escherichia coli 4) Staphylococcus aureus 5) Streptococcus pyogenes The Gram stain shows Gram positive cocci growing in clusters – typical of Staphylococcus aureus. This is the most likely organism to cause post-operative infection of prosthetic joints within the first 1-4 weeks following surgery.
  • 103. This 34 year old female presents with a one month history of weight loss, fatigue and this painless rash on her shin. Which of the following investigations would you choose for this patient? 1) ACE concentrations 2) Anti-TSH receptor antibodies 3) Chest X-ray 4) Fasting plasma glucose 5) OGD with duodenal biopsy This is pre-tibial myxoedema with slightly raised, pinkish, indurated patches usually on the fronts of the shins or dorsum of the foot and often associated with acropachy. A high titre of TSH receptor antibody is invariably associated with the dermopathy.
  • 104. This 42 year-old man was referred to hospital by his dentist. What is the diagnosis? 1) Addison's disease 2) Kaposi's sarcoma 3) Malignant melanoma 4) Mucor mycosis 5) Wegener's granulomatosis The picture shows characteristic purple lesions of Kaposi's sarcoma (KS) on the hard palate. The disease was originally described by Kaposi as a rare tumour in elderly men of Mediterranean origin ('classical Kaposi's sarcoma') in whom KS was usually found on the lower legs and feet. KS is now most commonly associated with HIV disease ('epidemic Kaposi's sarcoma'); it is seen predominantly in gay men. In HIV infection, KS lesions occur most commonly on the face; however lesions may be widely disseminated in skin, bronchial tree and G-I tract. KS can occur at any time during the course of HIV disease, irrespective of CD4 count; disseminated disease is commoner in late stage HIV disease/AIDS.
  • 105. This 22 year-old man presented to hospital with sudden opnset of dyspnoea, cough and haemoptysis. He was recently diagnosed HIV positive with a CD4 count of 580 cells/mm3. His chest x-ray showed a nodular infiltrate. What is the most likely diagnosis? 1) Cryptococcosis 2) Pneumocystis carinii pneumonia 3) Cytomegalovirus pneumonia 4) Kaposi's sarcoma 5) Varicella pneumonia The slide shows the typical rash of chicken pox (residue of calamine lotion can be seen on his lower back). Varicella pneumonia occurs in up to 20% of adults with chickenpox, appearing 3-5 days into the course of the illness. Symptoms include tachypnoea, cough, dyspnoea, and fever. Cyanosis, pleuritic chest pain and haemoptysis are common. CXR may show parenchymal nodules. The other HIV-related opportunistic infections listed, with the exception of KS, do not present with a rash and are associated with profound immunosuppression (CD4 < 200 cells/mm3 for PCP; CD4 < 50 cells/mm3 for the others). KS can present at any stage of disease.
  • 106. This 55 year old male is referred with a long history of infrequent shaving and low libido. Examination reveals the appearances as shown. Investigations reveal a testosterone of 6 nmol/l (NR 9-24 nmol/l), a LH of 22.1 (NR 3 - 10) and FSH of 28 (NR 3 - 10). Which of the following is the most appropriate investigation 1) Karyotype 2) MRI pituitary 3) Prolactin concentration 4) Short synacthen test 5) Thyroid function tests This patient has a hypogonadal appearance with gynaecomastia and the results demonstrate primary hypogonadism. This suggests Klinefelter's and karyotype would be the most appropriate investigation. Hyperprolactinaemia would cause Hypogonadotrophic hypogonadism. An MRI of the pituitary would be unhelpful as the pituitary is functioning normally.
  • 107. A 35 year-old woman with sickle cell disease was admitted to hospital with abdominal pain. This appearance was noted on fundoscopy. What is the diagnosis? 1) Branch retinal artery occlusion 2) Angioid streaks 3) Papilloedema 4) Central retinal vein occlusion 5) Lipaemia reinalis The slide shows angioid streaks, caused by breaks in Bruch's membrane. Angioid streaks may be associated with pseudoxanthoma elasticum, Ehlers-Danlos syndrome, sickle cell disease and Paget's disease.
  • 108. A 25-year-old man presents with vague chest pain and cough. His Chest X-ray, taken in the Emergency Department, is shown. What is the most appropriate treatment for this condition? 1) Amoxycillin/clavulanate + clarithromycin 2) Nebulized salbutamol 3) High-flow inspired oxygen 4) Intercostal chest drain insertion 5) Low molecular weight heparin The slide shows a small right apical pneumothorax. High-flow inspired oxygen is sufficient treatment; neither aspiration nor chest drain insertion is required.
  • 109. This 55-year-old man presented with a painful, swollen finger and a fever (37.7°C). What is likely to be found on microscopy of aspirated synovial fluid? 1) Bipyramidal crystals that exhibit strong positive birefringence under polarised light 2) Gram positive cocci in clusters 3) Needle-shaped crystals that exhibit strong negative birefringence under polarised light 4) Rhomboid crystals that exhibit weak positive birefringence under polarised light 5) Small, non-birefringent crystals visible only under electron microscopy An acute attack of monoarticular arthritis is typical of gout. Agonising pain is usually accompanied by joint swelling, redness and tenderness, often with a low-grade fever. Urate crystals are needle-shaped, and exhibit strong negative birefringence under polarised light microscopy. Calcium pyrophosphate crystals are described in answer (D); calcium oxalate crystals are described in (A); calcium hydroxyapatite crystals are described in (E). Septic arthritis caused by Staphylococcus aureus (option (B)) is unlikely to present as a monoarticular arthritis and would typically be associated with a higher temperature; the disgnosis can only be definitively excluded by joint aspiration, but the most likely diagnosis remains gout in this scenario.
  • 110. This 28 year-old man presented to hospital after becoming progressively more breathless over the preceeding day. He had developed a dry cough and reported expectoration of bright red blood. He gave a history of malaise and low-grade fever for 5 days. The rash (pictured) had appeared three days before presentation. What is the most likely diagnosis? 1) Varicella pneumonia 2) Goodpasture's syndrome 3) Meningiococcal septicaemia 4) Wegener's granulomatosis 5) Tuberculosis The slide shows the typical rash of chickenpox. Varicella pneumonia occurs in up to 20% of adults with chickenpox, appearing 3-5 days into the course of the illness. Symptoms include tachypnoea, cough, dyspnoea, and fever. Cyanosis, pleuritic chest pain and haemoptysis are common. CXR shows patchy shadowing. About 30 young adults die each year from varicella pneumonia and they are best managed on a High- Dependency Unit.
  • 111. With what diseases is this appearance associated? 1) Acromegaly 2) Poorly controlled diabetes mellitus 3) Pseudoxanthoma elasticum 4) Chronic lymphocytic leukaemia 5) Hypertriglyceridaemia 6) Paget's disease of bone 7) Acquired Immune Deficiency Syndrome 8) Retinitis pigmentosa 9) Cytomegalovirus infection 10) Closed angle glaucoma The slide shows angioid streaks, caused by breaks in Bruch's membrane. Angioid streaks may be associated with pseudoxanthoma elasticum, Ehlers-Danlos syndrome, sickle cell disease and Paget's disease.
  • 112. This lady's appearance has changed dramatically over the past week. She started a course of treatment four days ago. Blood was drawn prior to treatment (Results 1, below) and repeated urgently today (Results 2, below). Results 1 Results 2 T4 99 2 (NR 5-12 ug/dl) TSH <0.04 28 (NR 0.4 – 6.0 mU/l) What treatment has she received? 1) Propylthiouracil 2) Radioiodine 3) Carbimazole 4) Propranolol 5) Prednisolone The slide shows malignant exophthalmos: malignant refers to the rapidity of onset and threat to eyesight rather than association with malignancy. Radioiodine therapy can worsen thyroid- associated ophthalmopathy – patients with thyroid eye disease are generally treated with steroids for 1-2 weeks prior to starting radioiodine therapy. Treatment for malignant exophthalmos is rapid administration of steroids. Where sight is threatened, orbital decompression may be necessary.
  • 113. The following 12-lead ECG was obtained from a 48-year- old man following successful resuscitation from a community cardiac arrest. He had an extensive list of current and previous medication in his wallet. Which two of the drugs may have been the cause? 1) Amiloride 2) Amiodarone 3) Digoxin 4) Fentanyl patch 5) Haloperidol 6) Lignocaine 7) Ramipril 8) Salbutamol 9) Spironolactone 10) Temazepam A large number of drugs have been implicated in causing long QT interval and Torsade-de-pointes ventricular tachycardia (http://www.torsades.org/druglist.cfm). Digoxin causes a shortened QT interval. Amiloride, Ramipril and Spironolactone could lead to hyperkalaemia but it is hypokalaemia that is associated with Torsade- de-pointes VT. Lignocaine (and other Class 1b antiarrhythmics) shortens phase 2 of the cardiac cycle and therefore does not prolong the QT interval. Temazepam and Fentanyl have no effect on the QT interval.
  • 114. What organism has been cultured from a sputum sample from a 15 year old girl with a chronic cough and diarrhoea? 1) Pseudomonas aeruginosa 2) Staphylococcus aureus 3) Klebsiella pneumoniae 4) Haemophilus influenzae 5) Mycobacterium tuberculosis The culture plate shows a growth of Pseudomonas aeruginosa, characterized by the green colouration of the colonies - due to production of the pigment pyocyanin. The history is consistent with a diagnosis of cystic fibrosis: bronchiectasis associated with CF frequently results in recurrent infections with Pseudomonas.
  • 115. This 36-year-old lady was found to have hepatosplenomegaly. Which of the following are recognised complications of her disease? 1) Acute pancreatitis 2) Ischaemic heart disease 3) Polyarticular arthritis 4) Pathological bone fractures 5) Tetany
  • 116. The slide shows yellow papules (pingeculae) in the cornea; these are characteristic of Gaucher disease. Gaucher disease is inherited as an autosomal recessive disease. Disease is caused by a deficiency of the enzyme glucocerbrosidase, essentail for the metabolism of glycolipids. There are three types of Gaucher disease: (1) Type 1 (chronic non-neuropathic; adult Gaucher disease) (2) Type 2 (acute neuropathic; infentile Gaucher disease) (3) Type 3 (subacute neuropathic; juvenile Gaucher disease) Patients with all types of disease have hepatosplenomegaly and large glucocerebroside-rich cells (Gaucher cells) infiltrating the bone marrow. Type 2 (infentile Gaucher disease) carries the worst prognosis, with children seldom surviving beyond 2 years. Type 1 disease is the commonest, usually presenting in childhood with hepatosplenomegaly, but not uncommonly in middle- or old-age. Bone marrow replacement and hypersplenism result in anaemia and thrombocytopoenia. Pathological bone fractures and avascular necrosis of the femoral heads are not uncommon. Bony disease may be confined to the distal ends of the femurs, with formation of characteristic ‘Erlenmeyer flask’ shaped cysts. The skin may show a grey- brown discolouration, especially around the forehead, hands and pre-tibial regions. Characteristic yellow or yellow-brown papules (pingeculae) develop at the sclerocorneal junctions. Hyperlipidaemias manifest in the eye as corneal arcus senilis; hypertriglyceridaemia is associated with acute pancreatitis. Chronic hypercalcaemia can result in calcium deposition in the iris; hypercalcaemia is associated with tetany. Alkaptonuria usually presents in late- or middle-age with degenerative joint disease – ochronotic arthritis may resemble rheumatoid disease.
  • 117. This 45 year-old white female presented to her General Practitioner asking for advice regarding removal of these lesions for comsetic reasons. What additional investigation should be performed? 1) Biopsy of lesion for histology 2) Serum Cryptococcal antigen assay 3) HIV antibody test 4) Tzanck smear 5) Herpes virus serology Umbilicated, pearly papules 2-5mm in diameter. Seen in patients with advanced HIV/AIDS (CD4 count <200 cells/mm3). Caused by a pox virus. Commonly occur on face, especially near the eyelids; also occur on genitals and trunk. Treated with cryotherapy, liquid nitrogen or curettage.
  • 118. What is the diagnosis? 1) Addison's disease 2) Oral candidiasis 3) Lichen planus 4) Leukoplakia 5) Systemic sclerosis The slide shows lichen planus on the oral mucosa. Lesions are usually raised, flat- topped and violaceous with a fine white lace-like surface pattern (Wickham’s striae). Oral lesions are common.
  • 119. Which of the following is the most important in establishing the cause of these lesions? 1) Anti-streptolysin O titre 2) Skin biopsy 3) Chest X-ray 4) Erythrocyte sedimentation rate 5) Serum angiotensin converting enzyme (ACE) Erythema nodosum is shown in the slide. The commonest precipitant is a streptococcal infection. However, the commonest potentially serious causes (an therefore those that should be excluded first) include sarcoidosis and tuberculosis. A chest X-ray is an important investigation to exclude both of these causes.
  • 120. These are the hands of a 44 year old male who presents with abdominal pain. What is the cause for his abdominal pain? 1) Biliary colic 2) Diabetic ketoacidosis 3) Mesenteric ischaemia 4) Pancreatitis 5) Splenic infarction This patient has a type III hyperlipidaemia which is characterised by excess intermediate density lipoprotein (IDL) concentrations giving equally elevated total cholesterol and triglyceride concentrations. Typically, this condition is associated with premature atherosclerotic disease but also pancreatitis may occur.
  • 121. What is the most likely cause for this appearance? 1) Adenovirus 2) Cytomegalovirus 3) Herpes simplex virus 4) Pseudomonas aeruginosa 5) Staphylococcus aureus Dendritic ulcers are caused by Herpes simplex virus. Presentation is usually with pain, photophobia, blurred vision , conjunctivitis and chemosis. Steroid eye drops are contraindicated as they may induce massive amoeiboid ulceration and blindness. Diagnosis is by instillation of fluorescein eye drops which stain the ulcer (shown in slide).
  • 122. What does this ECG show on a 62 year old male who attends for a BUPA health check? 1) 1st degree heart block 2) Left bundle branch block 3) Left ventricular hypertrophy 4) Right Bundle branch block 5) Sinus rhythm This ECG shows a prolonged PR interval of 6 small squares 0.24s. The normal PR interval is 0.12 – 0.2 s. Causes of 1st degree heart block include increased vagal tone (such as in trained athletes), Ischaemic Heart Disease, rheumatic fever, hyperkalaemia, hypokalaemia, or drug therapy such as digoxin or beta-blockers. A long PR interval on the ECG may also be caused by structural abnormalities such as an Atrial Septal Defect. No treatment is usually required.
  • 123. The structure shown below was identified on microscopy of a sputum sample from a patient who presented with haemoptysis. What is the diagnosis? 1) Aspergillosis 2) Staphylococcal pneumonia 3) Goodpasture’s syndrome 4) Pneumoconiosis 5) Tuberculosis The slide shows the typical morphology of Aspergillus fumigatus.
  • 124. A 42-year-old Zimbabwean man was admitted to Casualty after becoming unconcious at Heathrow airport shortly after his arrival in the UK. His wife said that he had been unwell for several weeks, but had complained of a high fever associated with neck stiffness for the past 24 hours. Microscopy of his CSF is pictured. What organism is shown? 1) Listeria monocytogenes 2) Cryptococcus neoformans 3) Pneumocystis carinii 4) Toxoplasma gondii 5) Candida albicans Diagnosis is by demonstration of C. neoformans in the CSF – shown on an India ink stain: the thick polysaccharide capsule is highlighted around the cell (shown in slide). Cryptococcal meningitis is an AIDS- defining illness occurring when CD4< 50 cells/mm3 and may be associated with a pneumonitis. Cryptococcus can also cause papular skin lesions that resemble molluscum contagiosum. The disease is commoner in African populations.
  • 125. What is the diagnosis? 1) Central retinal artery occlusion 2) Cytmegalovirus retinitis 3) Retinal detachment 4) Toxoplasma retinitis 5) Central retinal vein occlusion The slide shows the typical appearance of central retinal vein occlusion. "Patients usually present with painless loss of vision and are found to have diffuse retinal hemorrhages in all four quadrants of the retina as well as dilated, tortuous veins. cotton-wool spots, disc edema, optociliary shunt vessels and neovessels might also be present. Multiple etiologies should be considered including: hypertension, glaucoma, optic disc edema, hypercoagulable states, vasculitis, drug-induced, and retrobulbar compression by tumors or grave's opthalmopathy."
  • 126. This 28 year-old man presented to hospital after becoming progressively more breathless over the preceeding day. He had developed a dry cough and reported expectoration of bright red blood. He gave a history of malaise and low-grade fever for 5 days. The rash (pictured) had appeared three days before presentation. What is the most appropriate treatment to start? 1) Intravenous benzyl penicillin + flucloxacillin pyrazinamide + ethambutol 2) High-dose corticosteroids 3) Co-amoxiclav + clarithromycin 4) Rifampicin + isoniazid + pyrazinamide + ethambutol 5) Intravenous aciclovir The slide shows the typical rash of chickenpox. Varicella pneumonia occurs in up to 20% of adults with chickenpox, appearing 3-5 days into the course of the illness. Symptoms include tachypnoea, cough, dyspnoea, and fever. Cyanosis, pleuritic chest pain and haemoptysis are common. In adults with pneumonitis, treatment with aciclovir is warranted.
  • 127. This 60 year-old man presented to his General Practitioner with a three-month history of proximal muscle weakness. He was a smoker of 20 cigarettes a day for 40 years. What is the diagnosis? 1) Dermatomyositis 2) Polymyalgia rheumatica 3) Cushing's syndrome 4) Myotonic dystrophy 5) Ectopic parathyroid hormone production Dermatomyositis is an autoimmune disease characterized by polymyositis and a skin rash. The classic purple (heliotrope) rash is seen on sun-exposed areas, especially the eyelids, nose, cheeks, forehead, knees, knuckles and around the nail beds. The rash may be pruritic. In older patients presenting for the first time with dermatomyositis an underlying malignancy should be considered. Malignancies most commonly associated with dermatomyositis include lung, ovary, breast and G-I.
  • 128. This 78-year-old man presented with a headache and blurred vision. An MRI angiogram showed a left posterior communicating artery aneurysm. Which of the following clinical signs would you expect to elicit? 1) Both pupils will react normally to light and accommodation 2) Left pupil will be unreactive to light or accommodation 3) Inability to blink on the left 4) Inability to elevate the globe on the left 5) Inability to frown 6) Inability to move left globe laterally 7) Left pupil will react to light but not to accommodation 8) Left-sided facial weakness 9) Loss of sensation over the left side of the face 10) Right-sided facial weakness
  • 129. The third (oculomotor) nerve nucleus complex lies in the midbrain. Motor neurones project to the ipsilateral medial rectus, inferior rectus, and internal oblique muscles and to the contralateral superior rectus. One central nucleus innervates levator palpebrae superioris bilaterally. Therefore a midbrain infarct that destroys the central nucleus will result in bilateral ptosis. Damage to the oculomotor nerve during its course results in ipsilateral ptosis; at rest, the globe is diverted downwards and laterally. The eye cannot be moved medially, superiorly or inferiorly. The effect on the pupil is variable, depending on the location of the lesion. Compression of the nerve (e.g. by tumour, posterior communicating or posterior cerebral artery aneurysms) results in an acute total (painful) third nerve palsy with a dilated unreactive pupil. Pupillary dilatation occurs early when the nerve is compressed since sympathetic nerve fibres that innervate the iris are carried on the outside of the nerve bundle. Pupillary sparing is characteristic of third nerve lesions caused by infarction in patients >50 years of age with diabetes or hypertension.
  • 130. What is the diagnosis in this patient with recurrent anaemia? 1) Ehlers-Danlos syndrome 2) Marfan syndrome 3) Pseudoxanthoma elasticum 4) Porphyria cutanea tarda 5) Tendon xanthomata The slide shows cigarette-paper (papyraceous) scarring of the skin. Ehlers- Danlos syndrome (EDS) is associated with joint laxity, hyperelasticity of skin, and fragility of skin and blood vessels. The scarring, shown here, is the result if minor trauma and subsequent slow healing. Eleven types of EDS are recognised, based on the mode of inheritance, severity of disease, and clinical manifestations.
  • 131. This genetic tree belongs to a family with hypophosphataemic vitamin D-resitant rickets. The abnormal gene is not re- introduced into the family by marriage or mutation. What is the chance that individual ‘A’ is affected? 1) 0% 2) 25% 3) 50% 4) 75% 5) 100% Hypophosphataemic vitamin D-resitant rickets is inherited in an X-linked dominant manner. Offspring of affected mothers, regardless of the children’s sex, have a 50% chance of acquiring the gene.
  • 132. A 24-year-old Irishman presented with a ten-week history of progressively worsening exertional dyspnoea and a dry cough. Auscultation of his chest revealed fine inspiratory crackles to the mid- zones. He was afebrile. His chest radiograph is shown below. A Heaf test was negative. Which of the following investigations will be most helpful in establishing a diagnosis? select three 9) HIV antibody test 10) Peak flow rate 11) Plasma lactate dehydrogenase (LDH) 12) Sputum culture 13) Transbronchial lung biopsy 14) Trial of steroids 15) 24 hour urinary calcium excretion 1) Abdominal ultrasound scan 2) Atypical serology 3) Bone marrow aspiration 4) Bronchoalveolar lavage 5) CD4 T-lymphocyte count 6) Cytoplasmic antineutrophil cytoplasmic antibody (cANCA) 7) C-reactive protein (CRP) 8) Erythrocyte sedimentation rate (ESR)
  • 133. The chest x-ray shows bilateral reticulonodular shadowing and bilateral hilar lymphadenopathy. The most likely diagnosis is sarcoidosis. Most cases of sarcoid present between 20 and 40 years of age with a slightly increased incidence in women. Prevalence varies amongst different ethnic populations; in Europe, sarcoid is commonest amongst caucasians – and has a significantly higher incidence in the Irish. A definitive diagnosis can only be made with biopsy evidence of a non-caseating granulomatous inflammatory process. Supportive evidence of sarcoid is provided by elevated serum angiotensin converting enzyme (ACE) levels, elevated 24-hour urine calcium excretion, increased uptake on gallium 67 scanning. Elevated inflammatory markers (ESR, CRP) are non-specific. Elevated serum calcium, though classical of sarcoid, is actually relatively rare. The main differential diagnosis in this case is HIV disease. Hilar adenopathy is common in HIV disease and CXR abnormalities are common. PCP frequently presents with a protracted history of worsening dyspnoea and dry cough with reticular shadowing on the CXR. Lymphopoenia is a common feature n both sarcoidosis and HIV. In this setting an HIV test is essential. CD4 counts cannot be used as a surrogate test of HIV infection since CD4 counts can be within the normal range in HIVseropositive individuals and is an ethically questionable means of trying to establish a diagnosis for which there is a definitive test.
  • 134. This 64 year old male presents with a six month history of weight loss and diarrhoea. Examination reveals the appearances as shown. Which of the following investigations would be most appropriate in this case? 1) Anti-endomysial antibodies 2) Barium enema 3) Sigmoidoscopy 4) Stool culture 5) Urine hydroxy indoleacetic acid This patient has a fixed facial flush and symptoms suggestive of Carcinoid syndrome. Therefore, Urine HIAA concentrations would be elevated. Other features that would be expected in carcinoid syndrome include hepatomegaly, right sided valvular lesions (as tumour products are metabolised by the lungs) and rarely pellagra. None of the other investigations listed would provide any diagnostic information.
  • 135. A 55 year old vagrant was brought to the Casualty department after being found unresponsive by a paramedic crew. His chest x-ray is shown. Select the three most important immediate investigations that should be performed? 1) CT scan thorax 2) Sputum for acid fast bacilli and mycobacterial culture 3) Carbon monoxide transfer factor 4) Mantoux test 5) Ventilation/perfusion (V/Q) scan 6) HIV antibody test 7) Sputum for viral culture & isolation 8) CT scan brain 9) Plasma antineutrophil anticytoplasmic antibody (ANCA) assay 10) Lumbar puncture 11) Peak expiratory flow rate 12) Bronchoscopy and transbronchial biopsy 13) Arterial blood gases 14) Intercostal chest drain insertion 15) Serum antibodies to atypical respiratory pathogens